FM COMAT

Pataasin ang iyong marka sa homework at exams ngayon gamit ang Quizwiz!

A 56-year-old white male presents to the office with low back pain. History reveals no recent trauma but the patient remarks that the pain was sudden and sharp and occurred while picking up a bag of groceries. Physical exam reveals localized paraspinal tenderness at T10 and 2+ patellar and Achilles deep tendon reflexes. Which of the following additional factors, if present, would warrant further diagnostic testing? A. less than 45-years-old B. pain greater than 3 weeks' duration C. prolonged use of glucocorticoids D.unexplained weight loss greater than 2.5% within one year E. vitamin D insufficiency

The correct answer is: C Low back pain is the second most common reason for clinician visits in the United States and requires an approach tailored to the specific patient. In a patient younger than 50 years old with no red flag symptoms or signs, conservative treatment with NSAIDs, local heat, and exercise is typically recommended for 4-6 weeks. Red flag symptoms include night pain, unrelenting pain at rest, neuromotor deficit, fever, loss of bowel or bladder control, trauma, osteoporosis, chronic corticosteroid use, or immunosuppression. The most common causes of low back pain include musculoligamentous strain, degenerative disc disease, and facet arthritis. Musculoligamentous strain is a tear/strain of muscle fibers/ligaments in the paraspinal muscles around the iliac crest/lower lumbar regions. The patient usually recalls an episode of bending/twisting or the back "giving way" when lifting a heavy object, with immediate onset of pain. Radiation of pain occurs across the low back and often to the buttock or upper thigh. In disc herniation, the nucleus pulposus extrudes through the annulus fibrosis and impinges on nerve roots, causing lower extremity radiculopathy, with most occurring at L5-S1 or L4-L5. Osteoarthritis is a common cause of chronic low back pain. Disc space narrowing and osteophytes can cause compression of nerve roots, causing lower extremity radiculopathy as well. Lumbar spinal stenosis is typically due to degenerative changes, with pain caused by activity and relieved by rest or spinal flexion (which increases the size of the canal). Vertebral osteomyelitis can be caused by hematogenous spread of any remote infectious process. Cauda equina syndrome occurs after spinal trauma or central lumbosacral disc herniation, which compress multiple sacral nerve roots. It is characterized by saddle anesthesia over the buttocks and perineum, with lower extremity weakness, and bowel or bladder dysfunction. It should be considered a surgical emergency. Vertebral compression is likely in this case, and is characterized by an acute back pain caused by a minor stress in elderly patients. Point tenderness at the level of the fracture with local radiation across the back and around the trunk and rarely into the legs is typical. In normal bone, this fracture requires severe flexion-compression trauma. In the elderly with osteoporosis, compression fractures can occur with minimal or no trauma. Multiple compression fractures can lead to severe kyphosis. Spontaneous vertebral body collapse, or pathologic fracture, is most commonly seen in elderly patients with severe osteoporosis, patients on long-term steroid therapy (with this answer choice), and cancer patients with lytic bony metastases or multiple myeloma.

A 68-year-old male with atrial fibrillation presents to the office for blood testing. History reveals that he takes warfarin 2 mg by mouth daily. Laboratory studies reveal a prothrombin time of 17.75 sec and INR of 2.32. The most appropriate recommendation at this time is to A. administer vitamin K B. continue with the current dosing schedule C. discontinue warfarin and start aspirin 81 mg PO D. increase warfarin to 4 mg PO daily E. skip the next dose and continue with 2 mg PO four times per week

B. continue with the current dosing schedule Atrial fibrillation (AF) is caused by multiple foci in the atria that fire continuously in a chaotic pattern, causing an irregular, rapid ventricular rate. Instead of contracting intermittently, the atria quiver continuously. Atrial rate is typically over 400/min but most impulses are blocked, so ventricular rate ranges between 75/min and 175/min. Patients with AF and underlying heart disease are at a markedly increased risk for adverse events, such as thromboembolism. Some common causes of AF include heart disease (coronary artery disease, myocardial infarction, hypertension, mitral valve disease), pulmonary disease, thyroid disease, systemic illness, stress, excessive alcohol intake, and pheochromocytoma. Patients typically present with fatigue, palpitations, dizziness, or angina. Acute AF in a hemodynamically unstable patient requires immediate electrical cardioversion to sinus rhythm. In acute AF in a hemodynamically stable patient, rate or rhythm control and anticoagulation are key. If AF is present for more than 48 hours or an unknown period of time, the risk of embolization during cardioversion is significant. Patients are anticoagulated 3 weeks before and 4 weeks after cardioversion. In chronic AF, rate control and anticoagulation are key. In lone AF (no underlying heart disease, patients under age 60), anticoagulation is not required, though aspirin may be appropriate. Other patients with chronic AF are treated with chronic anticoagulation. The CHADS2 and CHA2DS2-VASc are risk scores used to evaluate the risk of stroke and arterial embolization and to determine when the benefits outweigh the risks of antithrombotic prevention. Though there are several agents available on the market today, warfarin is the classic anticoagulant in patients who can tolerate it and who have greater risk of embolism versus bleed. Warfarin is dosed based on INR, which is typically tested monthly in stable chronic AF patients, with a goal INR of 2.0 to 3.0. You may refer to the laboratory document, which refers to the normal INR range as between 0.9 to 1.2 and for prothrombin time as between 12.0 and 14.0 sec. However, this is not a goal range for patients on warfarin for AF. This range, 2.0 to 3.0 must be committed to memory. Aspirin may be substituted for patients without risk factors for stroke or if contraindications to warfarin exist. This patient's INR falls within the range of 2.0 and 3.0. There are no signs of bleed, such as hematemesis, hemoptysis, melena, hematochezia, or hematuria. The patient should continue on this current schedule.

A 5-year-old female presents with fever and weight loss. Physical examination reveals generalized lymphadenopathy and splenomegaly. Laboratory studies reveal a platelet count of 120 x 103/mcL, a white count of 18 x 103/mcL, and an absolute neutrophil count of 500/uL. The most likely cause is A. anaplastic anemia B. CMV C. EBV D. lymphoblastic leukemia E. Streptococcal pharyngitis

The correct answer is: D Acute lymphoblastic leukemia is one of the most common childhood malignancies, typically affecting children ages 1 to 10 years old. Common presentation includes low grade fever, weight loss, malaise, pallor, petechiae and bone pain. Red flags on physical exam include generalized lymphadenopathy and hepatosplenomegaly. Laboratory studies are significant for anemia, thrombocytopenia, lymphocytosis with blast on peripheral smear, leukocytosis or leukopenia. Referral for immediate bone marrow biopsy and treatment is imperative. Answer A: Anaplastic anemia is a disease of stem cell failure that presents in adolescents and the elderly. Patient presents with fatigue, recurrent infections, mucosal bleeding and easy bruising. Laboratory findings include pancytopenia and hypocellular bone marrow on biopsy. Treatment with allogeneic stem cell transplant has and 80% long term survival rate. Answer B: CMV usually causes an asymptomatic infection or with mild flulike symptoms. After the initital infection, the virus remains latent and may reactivate. Most patients with CMV infection exhibit few clinical findings on physical examination. Primary CMV infection may be a cause of fever of unknown origin with pharyngitis, lymphadenopathy and splenomegaly. Symptoms, when apparent, develop 9-60 days after primary infection. Immunocompromised individuals are at risk for primary or reactivated symptomatic disease can affect almost every organ of the body. In patients with HIV infection, retinitis the most common manifestation of CMV disease. Answer C: Infectious mononucleosis is a clinical syndrome consisting of low grade fever, pharyngitis, and adenopathy. EBV patients may present with a maculopapular generalized non-puritic rash that rapidly disappears. Later physical findings include hepatomegaly, palatal petechiae, jaundice, uvular edema and splenomegaly. Lymphocytosis increases during the first few weeks of illness, and then gradually returns to normal. Thrombocytopenia is uncommon in EBV infectious mononucleosis. In the United States, approximately 50% of the population seroconverts before age 5 years, with much of the remaining seroconverting in adolescence or young adulthood. Answer E: The most common and important bacterial cause of pharyngitis is Streptococcus pyogenes (group A Streptococcus [GAS]). GABHS is also a common cause of erysipelas, cellulitis, and necrotizing fasciitis and has been reported as a cause of pneumonia, empyema, toxic shock syndrome, and lymphangitis. The vast majority of these manifestations do not occur in the setting of pharyngitis.

A 40-year-old male presents with fever and pain, swelling, and tenderness of the right great toe for the past 12 hours. Joint aspiration reveals negatively birefringent crystals. The most appropriate recommendation at this time is A. allopurinol B. colchicine C. indomethacin D. low purine diet E. probenecid

C. indomethacin Gout is a crystal-induced arthropathy caused by deposition of monosodium urate crystals. The most common joint affected is the first metatarsophalangeal joint but it can also affect other joints including ankle, foot, and knee. Joint aspiration shows needle shaped, negatively birefringent crystals. These crystals form secondary to high levels of uric acid. These high levels are typically due to underexcretion although it can be due to overproduction as well. These crystals cause an inflammatory response and an intense, sudden onset of pain. Gout flares are typically seen when there is an acute increase or decrease in uric acid levels. Acute gout attacks are treated primarily with pain relief with NSAIDs, indomethacin is a commonly used NSAID but any of them can be used.

A 65-year-old male presents to your clinic to discuss his poorly-controlled gastroesophageal reflux disease. History reveals that he has been taking a twice daily proton-pump inhibitor for over five years and that he now notes some difficulty swallowing. He also relates a long-standing history of smoking, but he has been unable quit. A fecal occult blood test is obtained and read as positive in the office. You note paravertebral hypertonicity from T5-9 that is tender to palpation. Which of the following is the most appropriate diagnostic test at this time? A. 24-hour pH probe monitoring B. barium swallow C. capsule endoscopy D. colonoscopy E. esophagogastroduodenoscopy

E. esophagogastroduodenoscopy Gastroesophageal reflux is actually a normal process that becomes a disease process when it causes macroscopic damage to the esophagus or causes symptoms that reduce quality of life. This occurs when the anti-reflux mechanism fails, either involving the lower esophageal sphincter, the crural diaphragm, or the part of the gastroesophageal junction below the hiatus. Patients may present with regurgitation of sour material in the mouth, along with heartburn (usually postprandial), dysphagia, chest pain, and hypersalivation. In severe cases, laryngitis, chronic cough, morning hoarseness, and aspiration may be seen. Typically, physical exam is normal. You may note TART changes in the thoracic spine as a result of viscerosomatic reflexes from the esophagus as this is the range of sympathetic innervation. Differential diagnoses to consider and exclude include: CAD, peptic ulcer disease, esophageal motility disorder, gastritis, infectious esophagitis, pill esophagitis, functional dyspepsia, and biliary tract disease. Mild, low-risk cases (without dysphagia, odynophagia, anemia, or weight loss) are usually diagnosed clinically, with successful treatment typically being diagnostic. Treatment includes lifestyle modification, specifically elevation of the head of the bed and dietary modification. Histamine-2 receptor antagonists (ranitidine, cimetidine, famotidine, nizatidine) work well for mild cases, but proton-pump inhibitors (omeprazole, lansoprazole, rabeprazole, esomeprazole, pantoprazole) are more effective, with maintenance therapy usually being required in that the patient will need to stay on the medication chronically. Surgery can be considered in patients whose symptoms persist despite optimal medical therapy. GERD is treated to prevent complications, including those related to prolonged acid suppression (pneumonia, enteric infections, vitamin B12 malabsorption), Barrett's esophagus, peptic stricture, asthma, and chronic cough, among many others. Indications for EGD (esophagogastroduodenoscopy or upper endoscopy) in patients with GERD include failure to respond to prolonged maintenance therapy, dysphagia, odynophagia, gastrointestinal bleeding, anemia, weight loss, and recurrent vomiting. Further, men older than 50 years with chronic GERD symptoms (greater than 5 years) and additional risk factors for Barrett's esophagus and esophageal adenocarcinoma (nocturnal reflux, hiatal hernia, elevated BMI, tobacco use, and intra-abdominal distribution of fat) should also have EGD. EGD can be used to evaluate for Barrett's, erosions, strictures, or ulcers. Many aspects of this case are concerning, including his age, his duration of symptoms, dysphagia, malaise, and continued smoking.

Vaccine for A 25-year-old healthcare professional presents to the primary care physician for a pre-employment examination.

Hepatitis B vaccine is an inactivated vaccine series with 3 injections (0 month, 1 month and 6 months). If there is a questionable history of vaccination, hepatitis B titers can be checked prior to administering hepatitis B vaccination series. Mode of transmission of hepatitis B includes sexual intercourse and blood or blood contaminated products. Therefore, it is highly recommended for adults with increased risk of sexual transmission of infections including men who have sex with men, multiple sexual partners, sex partner of a patient with hepatitis B and upon evaluation of any sexually transmitted infection. Other indications include all health care workers, comorbidities (diabetes mellitus, HIV infection, chronic liver disease, end-stage kidney disease), and travelers to countries with endemic hepatitis B. As per CDC recommendation, hepatitis B vaccination series are recommended for all age groups, since it is one of the leading causes of chronic liver disease and cirrhosis worldwide. Therefore, all adults with high risk sexual behavior including men who have sex with men should be recommended a hepatitis B vaccination

Vaccine for An 18-year-old male who recently started college, living on campus.

Meningococcal vaccination is also an inactivated vaccination typically administered as one shot. Most common recommendation is in teenagers starting college due to living in closed spaces like dormitories. For similar reasons, it is also recommended in military personnel as well. Other indications include patients with asplenia and complement deficiencies as well as HIV infection patients. Therefore, patient's history of joining the military makes it the most likely vaccination to be administered.

A 65-year-old female with past medical history of diabetes presents to the clinic complaining of 3 month history of painful feet. She states her pain is intermittent, worse in the evening, and of burning nature. Her pain is mainly located in her feet bilaterally but now also spreading to her hands as well. It is also associated with some numbness and tingling in her hands and feet as well. She has tried tylenol and ibuprofen with no relief. Her home medications include metformin and insulin twice a day. The most appropriate management of her pain is A. acetaminophen with oxycodone B. amitriptyline C. capsaicin D.topical lidocaine E. tramadol

The correct answer is: B Diabetic neuropathy typically presents as painful numbness and tingling in a "stocking-glove" pattern. Although tight glycemic control is one of the key steps in management of diabetic neuropathy, pain control is very essential part of management as well. Tricyclic antidepressants (amitriptyline) or anticonvulsants (pregabalin, gabapentin) are appropriate for painful diabetic neuropathy. TCAs such as amitriptyline and desipramine are most often used. TCA's increase the synaptic concentration of serotonin and/or norepinephrine in the central nervous system by inhibition of their reuptake by the presynaptic neuronal membrane pump. Common side-effects of tricyclic antidepressants include dry mouth and somnolence. Pregabalin is an alpha 2 delta ligand which acts as a presynaptic inhibitor of the release of excitatory neurotransmitters like glutamate and substance P. It is FDA approved for treatment of diabetic neuropathy as many studies have shown significant reduction in pain compared to placebo. Side effect profile includes drowsiness, dizziness, confusion. Answer A: There is no role of opioids as a first line drug for management of painful diabetic neuropathy. Although, initial treatment can include over the counter analgesics and topical capsaicin cream, there is no role for opioids unless management with TCA or selective norepinephrine reuptake inhibitors have failed. Answer C: Capsaicin cream is a topical anesthetic cream which is made out of a naturally occurring component of hot peppers. Many studies have shown statistically significant reduction in local burning and pain compared to placebo, however, it is not considered first line therapy for painful diabetic neuropathy due to side effect profile and limited results. Many patients are unable to tolerate it due to local burning produced by the application of capsaicin. Therefore, making it an incorrect answer choice. Answer D: Topical lidocaine patches can be helpful, however, they are not first line therapy. Lidocaine works by blocking both the initiation and conduction of nerve impulses by decreasing the neuronal membrane's permeability to sodium ions, which results in inhibition of depolarization with resultant blockade of conduction. Answer E: Tramadol is a weak opioid drug which has inconclusive data regarding improvement in pain in diabetic neuropathy. It can be used in patients whose pain is refractory to management with TCAs or pregabalin; however, it is not the first line therapy making it an incorrect answer choice.

A 16-year-old male presents for a routine wellness examination. Physical examination reveal open and closed comedones without evidence of surrounding inflammation on his face and upper back. He has no other past medical history and his developmental milestones have been met in an appropriate timeframe. Which of the following is the most appropriate initial treatment? A. dietary modification B. oral minocycline C. oral isoretinoin D. topical clindamycin E. topical tretinoin

The correct answer is: E This patient has comedonal acne, which is apparently being diagnosed for the first time. He has had no treatment up to this point, and the presentation of his acne does not sound very severe (i.e. absence of current scarring or inflammation). Topical retinoids, such as topical tretinoin, are appropriate for first-line treatment of inflammatory and/or comedonal acne. These agents work by decreasing obstruction in the tissue surrounding the hair follicles, thereby promoting adequate drainage and clearance of debris and bacteria. Answer A: There are few clinical studies that have looked at the augmentation of dietary intake (either certain foods or broad food groups) to improve the control of acne. Some research has been done to see if acne exacerbations are related to certain foods; however, they have failed to show a distinct link to these breakouts and food. Thus dietary and lifestyle modification would be of no utility in this patient. Answer B: Oral antibiotics, such as minocycline (a tetracycline) are well-studied in the treatment of infectious and inflammatory acne. It is a useful adjunct, but should not be used as initial or monotherapy due to increased rates of resistance in Propionebacterium acnes. Erythromycin has also been used, but is more likely to succumb to resistance patterns than tetracyclines. Minocycline has been known to cause vaginal candidiasis, photosensitivity, skin mispigmentation, and (less commonly) autoimmune hepatitis, drug-induced lupus, and serum-sickness like syndromes. Other antibiotics, such as trimethoprim-sulfamethoxazole have been used but are less studied. Answer C: Oral isoretinoin has been approved for the treatment of severe recalcitrant acne. This patient has a milder form of acne that has not been treated to this point, and thus oral isoretinoin is not an option for this patient as first-line therapy. Isoretinoin is a known teratogen and has also been linked to mood disorders, depression, suicidal ideation and suicide attempts. Answer D: Topical antibiotics, such as clindamycin, can be effective and are very well-studied. However, as with oral antibiotics, resistance patterns discourage its use as monotherapy. Thus topical clindamycin would not be the best initial choice for this patient.

A 46-year-old male presents for a routine wellness examination. History reveals that he has diabetes mellitus type 2 treated with an oral medication and that his most recent HbA1C was 7.8%. On physical exam you note paravertebral hypertonicity from T6-9 and tenderness to palpation around this region. There is also a tender nodule noted one inch lateral and superior to the umbilicus. Laboratory studies are obtained and reveal the following: Na+ 139 mEq/L K+ 4.5 mEq/L Cl- 98 mmol/L HCO3- 22 mEq/L BUN 35 mg/dL Cr 1.8 mg/dL Glucose 207 mg/dL Which of the following medications is CONTRAINDICATED in this patient? A.acarbose B. insulin C. metformin D. nateglinide E. pioglitazone

The correct answer is: C Metformin is contraindicated in renal insufficiency, especially in those older than 80 with poor renal function. It should not be used in male patients with a creatinine greater than 1.5 or females with creatinine greater than 1.4. It works by decreasing hepatic gluconeogenesis and increasing the effectiveness of insulin. It is commonly the first line drug in type 2 diabetes because it does not cause hypoglycemia and is probably the oral agent that this patient is taking. Side effects include gastrointestinal upset, lactic acidosis, and decreased vitamin B12 absorption. It is excreted 100% in the urine. Patients with renal impairment as this patient has are more likely to experience the side effects including lactic acidosis. Lactic acidosis causes an anion gap metabolic acidosis, and this patient has an elevated anion gap of 19 and if an arterial blood gas were drawn could possibly be acidotic. Anion gap is calculated simply by subtracting the sum of the anions from the cations, i.e. [Na+] − ([Cl-] + [HCO3−]). Potasium is generally omitted from the calculation because the extracellular potasium concentrations are very low. Other abnormalities in this patient's lab results include a slightly elevated BUN, normal should be between 6-20 mg/dL, elevated creatinine as discussed above, and an elevated glucose level indicative of this patient's diabetes. Aside from elevated creatinine, you can use clues from your structural exam to help identify renal dysfunction when determining which medication to initiate. In this case we have TART changes from T6-9 representing viscerosomatic reflex from the kidneys, as well as an anterior chapman point for the kidney, one inch lateral and superior to the umbilicus. Answer A: Acarbose is an α-glucosidase inhibitor. Its mechanism of action is by inhibiting the enzyme that degrades starch and disaccharides it decreases the gastrointestinal absorption of carbohydrates. It can be used as monotherapy in patients who have adequate diet control of their diabetes but it is most often used as an adjunct to other oral hypoglycemic drugs. Side effects include diarrhea and flatulence. Answer B: Insulin is used in type I diabetes mellitus and in later stages of type 2 diabetes mellitus. It is given subcutaneously and does not have an oral formulation. Caution should be taken in patients who have renal impairment, and it should be dosed based on creatinine clearance. It is not contraindicated with renal insufficiency. Answer D: Nateglinide is in the drug class of meglitinides with stimulate insulin release from pancreatic β-islet cells. It works similarly to sulfonylureas but unlike sulfonylureas it can be used in patients with renal insufficiency. It is significantly more expensive than sulfonylureas. It is usually used as an adjunct to metformin. Major adverse effect is hypoglycemia. Answer E: This is in the drug class thiazolidinediones and works by increasing insulin sensitivity. It is usually used as an adjunct to other drugs. Side effects include weight gain and an increase in serum LDL. It has no restrictions in renal impairment.

A 44-year-old female presents with right knee pain and swelling. History reveals that she scrubbed three stories of her house and was on her knees for long periods of time. Physical examination reveals localized swelling and tenderness present anterior to the kneecap which is moderately warm to the touch. The most likely diagnosis is A. patellar tendinitis B. pes anserine bursitis C. prepatellar bursitis D. semimembranosis bursitis E. suprapatellar bursitis

The correct answer is: C Prepatellar bursitis is swelling and inflammation of the bursa anterior to the patella. The bursa ordinarily acts to enhance gliding of tissue over the patella. However, inflammation can occur due to trauma, as a result of a fall, or by direct pressure and friction of repetitive kneeling. Prepatellar bursitis is also known as "housemaid's knee." Physical examination will reveal pain and swelling of the anterior knee. The knee may also be warm to the touch. The next step is to aspirate the inflamed bursa and send the fluid for gram stain and culture in order to distinguish between septic versus aseptic. Treatment is with NSAID's and compressive wrapping. Answer A: Patellar tendinitis, also known as "jumper's knee," is an activity related anterior knee pain that more commonly occurs in adolescents and young adults. It results in insidious onset of focal anterior knee pain at the inferior border of the patella. Early on, it causes pain following activity. In the later phases, it causes pain during activity. Physical examination is key, as patient will have focal tenderness located at the inferior pole of the patella at the origin of the patellar tendon. Radiographs are usually normal, or may show an inferior traction spur in chronic cases. Treatment is with rest, ice, activity modification, and NSAID's. Answer B: Pes anserine bursitis is an inflammatory condition of the bursa located at the medial side of the knee. The pes anserine, also known as the "goose's foot," is located at the anteromedial proximal tibia. It is formed by the confluence of three tendons onto the tibia, the sartorius, gracilis, ad semitendinosus tendons. This bursa may become inflamed and cause medial sided knee pain. Risk factors include tight hamstrings, direct trauma, valgus knee deformity, pes planus or flat foot, obesity, and sporting events that require side to side movement. It is more common in overweight females due to the different angulation of the female knee, which puts more pressure over the pes anserine tendons. Thus, pes anserine bursitis is more commonly the result of an abnormal gait. Treatment is with activity modification, ice, and NSAID's. Answer D: Semimembranosus bursitis is inflammation of the bursal sac that is location in the posterior knee between the semimembranosus muscle and the medial head of gastrocnemius. Inflammation of this bursal sac results in what is known as a Baker's cyst. It is more common in elderly patients, and associated with arthritis of the knee. Patient's will present with pain located in the posteromedial aspect of the knee. Physical examination will reveal point tenderness to the semimembranosus tendon which inserts onto the medial proximal tibia. Initial treatment involves activity modification, ice, nonsteroidal anti-inflammatory drugs, and physical therapy. If this persists greater than a couple months, a corticosteroid injection at the tendon insertion may be warranted. Answer E: The suprapatellar bursa is a fluid filled sac that is located beneath the patella and the quadriceps femoris tendon. It is very vulnerable to trauma and repeated microtrauma including falls onto the knee, overuse injuries from running or crawling on knees. Patients with suprapatellar bursitis will present with anterior knee located above the patella that may radiate proximally into the distal quadriceps muscles. Patients may have trouble bending their knee when walking down stairs. Physical examination will reveal point tenderness in the anterior knee at the superior pole of the patella. There may be the presence of swelling and edema in the suprapatellar region. Passive knee flexion and resisted extension of the knee will reproduce the pain. Initial treatment consists of rest, NSAIDs, ice, and a knee brace to prevent further trauma.

A 68-year-old male with past medical history of hypertension, diabetes mellitus type II, benign prostatic hyperplasia, gastroesophageal reflux, and chronic low back pain is undergoing a computed tomography scan with IV contrast for a suspected parapneumonic empyema. Which of the following medications must be discontinued prior to the study? A. famotidine B. hydrochlorothiazide C. metformin D. oxycodone/acetaminophen E. tamsulosin

The correct answer is: C There are three main kinds of contrast available: intravenous (IV), oral (PO), and PR (per rectum). IV contrast can be either gadolinium for MRI or iodinated contrast for CT. Oral contrast is dilute iodinated contrast. Barium can be used in patients with allergic reactions to iodinated contrast and these patients generally should be premedicated for oral iodinated contrast, specifically if they have had a history of moderate or severe allergy. When able to be administered, IV contrast is preferred because it improves tissue contrast, allowing one organ to be differentiated from those nearby. Masses and inflammatory processes are also better visualized. Thus, in this patient with suspected empyema, we would prefer IV contrast. Even if empyema were not visualized, this imaging might show some other offending process. There are two main contraindications for the administration of iodinated IV contrast: contrast-induced nephropathy and allergy to iodinated contrast. A creatinine of less than or equal to 1.5 mg/dL and no evidence of acute kidney injury are prerequisites for IV contrast administration. Even patients with a history of moderate anaphylactoid reaction to iodinated contrast (hives, facial swelling, itching) can feasibly receive it if the risks outweigh the benefits, the patient is monitored closely, and they receive premedication (steroid and diphenhydramine). Patients should not receive IV contrast if they have had a severe reaction, including laryngeal edema, cardiopulmonary arrest, or profound hypotension. Even patients who have only one kidney or who are transplant recipients can receive it, though the volume of contrast they receive will be adjusted accordingly. Metformin, an oral biguanide hyperglycemic therapy for diabetes mellitus type 2, is primarily excreted by the kidney. In patients with severe renal impairment, intravascular accumulation may occur after radiological contrast media administration, and this may precipitate lactic acidosis. This presents with vomiting, diarrhea, and somnolence, and is a potentially fatal complication. Lactic acidosis is extremely rare in diabetics receiving metformin if the patient has normal renal function before contrast administration. However, it remains a risk. Patients taking metformin should have the medication withheld for 48 hours following contrast administration and should have their renal function re-checked prior to re-starting the medication. These reactions are more common in women and not dose-dependent. Consider also that many patients take diabetes medications that include metformin in combination with another medication and these need to be recognized and discontinued as well.

A 60-year-old female presents to her primary care physician with hip pain which she reports is worse at the end of the day. Vital signs are obtained as follows: Height 1.32 m (60 in) Weight 104 kg (230 lbs) Temperature 37.6ºC (99.8ºF) Blood Pressure 135/85 mmHg Pulse 88/min Respiration 18/min O2 saturation 98% Physical examination reveals decreased range of motion in her affected joints and deformity of her distal interphalangeal joints on her hands. Structural examination reveals a positive seated flexion test on the left, a sacrum with a deep left sulcus, a left inferior lateral angle that is inferior and posterior, and springing at the left sulcus. Most likely diagnosis is:

Osteoarthritis is a chronic joint disease caused by articular cartilage degeneration which results in decreased joint space. It most commonly affects weight bearing joints including hips and knees but can also affect hands, wrists, shoulders, and vertebrae. Risk factors include increased age, obesity, family history, and previous joint trauma. Patients will present with pain and joint stiffness that gets worse with activity and weight-bearing activity and is usually relieved by rest. Patients will have decreased range of motion. Osteoarthritis commonly affects the distal interphalangeal joints of the hands causing deformities called Heberden's nodes. Radiographs of the affected joints will demonstrate joint space narrowing, osteophytes, subchondral sclerosis and subchondral cysts. Osteoarthritis is generally non-inflammatory and treatments includes weight loss, lifestyle modification, analgesics, corticosteroid injections, and as the disease progresses possibly joint replacement. Osteoporosis is not likely in this patient because of her age and her weight. Screening for osteoporosis is recommended to start at age 65. Bone density decreases with a decrease in the amount of estrogen that women experience with menopause. The typical osteoporosis patient is a thin white female. This patient's weight increases load bearing on her bones which leads to increase in bone mineralization to compensate for the extra stress.

Vaccine for A 66-year-old male presents to the primary care physician after placement of new cochlear implants.

Pneumococcal vaccine is an inactivated vaccination recommended in all adults of age 65 years and older. In adults age 19 - 64 years of age, specific indications include diabetes mellitus, chronic pulmonary disease (asthma and COPD), cigarette smoking, cochlear implant, alcoholism, immunocompromised patients (HIV, multiple myeloma, chronic corticosteroids) and asplenia to list a few. Patients who are vaccinated prior to age 65 years, receive a booster dose at age 65 or 5 years after initial vaccination, whichever is later. Therefore, given the history of cochlear implant, pneumococcal vaccine is the most likely diagnosis.

A 79-year-old female presents to the office for a routine wellness examination with complaints of chronic back pain. History reveals that she has gotten shorter over the past decade, but she denies any history of acute fracture. A DEXA scan is performed, revealing a T-score of -2.56. The most appropriate recommendation A. calcium 400 mg with vitamin D 400 IU B. calcium 1,000 mg with vitamin D 600 IU C. calcium 1,200 mg with vitamin D 800 IU D. calcium 2,500 mg with vitamin D 2,000 IU E.vitamin D 50,000 IU once weekly

The correct answer is: C The first step in this question requires knowledge of DEXA scores and how to interpret them. A T-score of less than 2.5 gives this patient a diagnosis of osteoporosis. Remember, scores between -1 and -2.5 diagnose osteopenia, and scores greater than -1 indicate normal bone density. This patient's chronic low back pain and decrease in height are important and indicate the possibility of subclinical, small vertebral fractures. However, without sign of overt fracture, she does not qualify for a diagnosis of severe osteoporosis. The treatment of osteoporosis consists of lifestyle changes and pharmacologic therapy. All postmenopausal women with osteoporosis should receive adequate counseling regarding exercise, smoking cessation, fall prevention, and avoidance of heavy alcohol use. Affected patients should avoid medications that increase bone loss, including glucocorticoids. Postmenopausal women with osteoporosis should ensure that their total calcium intake (diet and supplements) approximates 1200 mg/day. If the diet is insufficient then supplemental elemental calcium of 500-1000 mg/day in divided doses is recommended. Women with adequate dietary intake do not require calcium supplementation, as it will be unhelpful to the disease process and may increase the incidence of cardiovascular disease. Women should also ingest a total of 800 international units of vitamin D daily. Further, women should exercise for at least 30 minutes three times a week in order to treat and prevent osteoporosis. Many types of exercise are effective, though the inclusion of weight-bearing exercise is most important for its ability to affect bone architecture. Smoking cessation is recommended because smoking cigarettes accelerates bone loss. Pharmacologic therapy for postmenopausal women with a history of hip or vertebral fracture or with osteoporosis based on DEXA scan should also be started. For most postmenopausal women with osteoporosis, oral bisphosphonates are considered to be first-line therapy and these include alendronate and risedronate. Intravenous zoledronic acid and intravenous ibandronate are also available. Raloxifene can be used in patients who cannot tolerate bisphosphonates or those with increased risk of invasive breast cancer. Parathyroid hormone (teriparatide) is an anabolic agent that can be used in postmenopausal men or women with severe osteoporosis and fracture or in patients for whom other therapies fail.

A vaccine for 26-year-old female who just had a positive pregnancy test at home presents to her primary care physician and asks which vaccination carries the highest risk of birth defects.

Varicella vaccine is a live, attenuated vaccination. Varicella vaccine has been part of the children immunizations since 1996. It is recommended in all persons born after 1980 unless there is a physician documented evidence of either varicella or varicella vaccination. Since it is a live vaccine it is used with caution in immunocompromised patients. It is contraindicated in pregnancy and women who become pregnant within one month of vaccination should be counseled regarding the risk of birth defects from a live vaccine. Since all the other vaccinations listed above are inactivated vaccines except varicella, varicella has the highest possible risk of birth defects

A 7-year-old male presents after sustaining a bite wound to his left arm. History reveals that he was wrestling with his brother when the injury occurred. Which of the following antibiotics is most appropriate at this time? A. amoxicillin / clavulanate B. cephalexin C. ciprofloxacin D. clindamycin E.erythromycin

The correct answer is: A A gram stain and aerobic and anaerobic cultures should be obtained for all infected wounds before any therapy is given. Wounds should be copiously irrigated, surgically debrided, and diagrammed or photographed (or both). Immobilization of the affected area, including splinting if necessary, and elevation should be instituted. Empirical antimicrobial therapy should be based on the gram stain or knowledge of the susceptibility of the oral flora. Patients who present early with uninfected wounds should also be given antimicrobial therapy of shorter duration and may be considered for outpatient management. Amoxicillin/clavulanic acid or penicillin plus a penicillinase-resistant penicillin or cephalosporin should be used. Answer B: Cephalexin inhibits bacterial cell wall synthesis by binding to one or more of the penicillin-binding proteins which in turn inhibits the final transpeptidation step of peptidoglycan synthesis in bacterial cell walls, thus inhibiting cell wall biosynthesis. It is effective for Streptococcal or Staphylococci infections, including penicillinase-producing Staphylococci. Answer C: Ciprofloxacin inhibits relaxation of DNA, inhibits DNA gyrase in susceptible organisms, and promotes breakage of double-stranded DNA. It may be used for bite wounds due to its activity against Eikenella, however, it is typically used in combination with metronidazole or clindamycin. Answer D: Clindamycin suppresses protein synthesis by binding to 50S ribosomal subunits. It may be used in combination with ciprofloxacin in treating bite wounds for anaerobic coverage, however, it lacks activity against Eikenella corrodens when used alone. Answer E: Erythromycin has poor activity against Eikenella and should be avoided.

A 3-year-old male with past medical history allergic rhinitis presents to the emergency room with a fever and ear pain. History reveals he was previously diagnosed as a upper respiratory infection at an urgent care clinic two weeks ago that has since resolved and started swimming lessons this week. Vitals reveal a temperature of 40ºC (104ºF). Physical examination reveals an irritable child in mild distress. Evaluation of the skin is negative for rashes or lesions, the nasal and oral mucosa is pink and moist without pharyngeal exudation, the chest is clear to auscultation with a heart rate of 120/min, and the abdomen is soft and non-tender. Otoscopy demonstrates a tympanic membrane that is pale yellow, mildly injected, and immobile with insufflation of air. The most likely diagnosis is A. acute otitis media B.cholesteatoma C. otitis externa D. otitis media with effusion E. tympanosclerosis

The correct answer is: A Acute otitis media (AOM) is typically a complication of Eustachian tube dysfunction that occurred during an acute viral upper respiratory infection (URI). S. pneumoniae, H. influenzae, and M. catarrhalis are the most common organisms implicated. Risk factors include young age, allergies, craniofacial abnormalities, exposure to cigarette smoke and environmental irritants, participation in group day care, family history of recurrent AOM, , GERD, immunodeficiency, history of bottle feeding, pacifier use, and URI. AOM is defined by moderate to severe bulging of the tympanic membrane (TM), new onset of otorrhea not caused by otitis externa (OE), or mild bulging of the TM associated with recent onset of ear pain (less than 48 hours) or erythema. Children without objective evidence of middle ear effusion (MEE) should not be diagnosed with AOM, since this leads to antibiotic resistance through unnecessary antibiotic treatment. Importantly, AOM should be differentiated from otitis media with effusion (OME). OME is middle ear effusion w/o acute symptoms. Inflammation with bulging of the TM on otoscopy is highly predictive of AOM. Analgesics should be considered for ear pain, fever, and irritability, helping especially at night to treat disrupted sleep. Antibiotic therapy should be deferred in patients who are less likely to benefit from antibiotics. Those children with AOM who are 6 months or older with severe signs or symptoms (moderate or severe otalgia, otalgia > 48 hours, temperature > 102.2ºF) and those younger than 2 years with bilateral AOM should be prescribed antibiotics. Children with mild symptoms can be observed if they are between 6 and 23 months and have unilateral AOM. Those with mild symptoms who are two years or older can be observed regardless of unilateral or bilateral involvement. If observation is chosen and symptoms persist for more than 48 to 72 hours, a follow-up visit should be scheduled to re-examine the ear and to consider antibiotics or change in antibiotic therapy. High-dose amoxicillin is the initial treatment of choice if antibiotics are required. Cefuroximine is to be used in penicillin-allergic patients. Second-line treatment is high-dose amoxicillin/clavulanate.

An 18-month-old male presents with a 4 day history of rhinorrhea, mild fever, and nocturnal cough. Physical examination reveals scattered wheezes bilaterally. An AP and lateral chest radiograph is obtained and read as negative. The most likely diagnosis is A. bronchiolitis B. epiglottitis C. foreign body D. laryngotracheobronchitis E.typical pneumonia

The correct answer is: A Bronchiolitis is an inflammatory illness of the small airways that primarily affect children under the age of 2 years old. History will typically consist of low-grade fever, rhinorrhea (runny nose), cough, and may also have periods of apnea. This can be a potentially fatal condition that may progress to respiratory failure. The most common cause is RSV. Diagnosis is clinical and may have the work up which includes chest x-ray and for boards ELISA of nasal washings for RSV. The chest x-ray may be negative or hyperinflated with interstitial infiltrates, and/or atelectasis. Mild disease can be treated outpatient with fluids, nebulizers and possibly oxygen. If the patient has respiratory distress the patient should be hospitalized and check an oxygen saturation. Warning signs to look for is oxygen saturation < 92%, < 3 months of age, underlying cardiopulmonary disease, and/or unreliable parents. Inpatient treatment may consist of contact isolation, hydration, oxygen supplementation and nebulizers. Answer B: Epiglottitis is an infection of the supraglottic structures. The most common pathogens are H. influenzae type B; however due to immunizations this is trending down and now Streptococcus spp., non-typable H. influenzae, and viral agents are becoming more common. Physical exam consists of high fever, dysphagia, drooling, and muffled voice. The patient may be seen in what is called the sniffing dog position (patient sitting up, leaning forward with their chin protruding. This is to maximize air entry. Do not examine the throat unless there is a very experienced physician for difficult intubations present (anesthesiologist). Lateral x-rays demonstrate a thumbprint sign. Treatment consists of endotracheal intubation, in the operating room, or tracheostomy, and iv antibiotics (ceftriaxone or cefuroxime). Answer C: Foreign body would consist of sudden onset of difficulty breathing. The history may reveal playing with small toys or objects that can easily be swallowed. Physical exam may reveal stridor. Chest x-ray would reveal a foreign body within the airway. Treatment is removal with bronchoscopy. Answer D: Laryngotracheobronchitis (croup) is an inflammatory disease of the larynx. This is most commonly caused by parainfluenza virus type-I. History usually reveals URI symptoms, low grade fever, mild dyspnea, inspiratory stridor, hoarse voice and the classic characteristic barking cough. Chest x-ray typically reveals narrowing in the subglottic region (steeple sign). Treatment consists of: mild - cool mist therapy, moderate - oral corticosteroids, severe - nebulized racemic epinephrine. The key differences of laryngotracheobronchitis from bronchiolitis is the barking cough, inspiratory stridor and steeple sign on chest x-ray. Answer E: Typical pneumonia is a clinical diagnosis. The most common pathogen is S. pneumoniae. Remember that with typical pneumonia there is a lobar consolidation seen on chest x-ray. History consists of fever, productive cough, and dyspnea. Physical exam reveals rhonchi and egophony in a lobar segment, not diffuse. Chest x-ray reveals a lobar consolidation. Treatment consists of a third generation cephalosporin with a macrolide (ceftriaxone and azithromycin), or a respiratory fluoroquinolone (levofloxacin). Do not use fluoroquinolones in children due to achilles tendon rupture.

A 55-year-old male presents to the office for a routine wellness examination. Past medical history is unremarkable. Vitals reveal the following: Heart rate Blood pressure 76 166/84 81 172/91 77 156/84 69. 183/95 73 164/102 78 174/98 Physical examination reveals heart regular rate and rhythm, lungs are clear to auscultation bilaterally, abdomen is soft and non-tender, there is no peripheral edema noted. Fundoscopic examination will most likely reveal A. cotton-wool patches B. Drusen C. hard exudates D. microaneurysms E. Roth spots

The correct answer is: A Cotton-wool patches, also known as soft exudates, are white ovoid lesions with irregular borders seen on retinal examination as a result of infarcted nerve fibers. Cotton-Wool Patches along with A-V nicking are commonly seen in patients with hypertension. There is no specific treatment for Cotton-Wool Patches, but it is recommended to treat the underlying hypertension to prevent further hypertensive damage to the retina. In this patient, the vitals review indicates stage II hypertension. Stage II hypertension is defined as systolic blood pressure greater than 160 mmHg or a diastolic blood pressure greater than 100 mmHg on two separate occasions. Therefore one should be looking for a retinal exam finding common in hypertension. Of the provided choices, cotton-wool patches is the most appropriate choice. Patients presenting with stage II hypertension often need to be treated with two antihypertensives to adequately control blood pressure to the goal blood pressure of < 140/90 mmHg. The initial choice of antihypertensive may be an ACE-inhibitor, diuretic, or calcium channel blocker.

For each numbered item (patient presentation), select the one heading (treatment) most closely associated with it. Each lettered heading may be selected once, more than once, or not at all A. citalopram B. donepezil C. memantine D. omega-3 fatty acids E. selegiline An 82-year-old female with a past medical history of hypertension, hyperlipidemia, and mild dementia presents to her primary care physician with her daughter. Her daughter is worried because the patient refuses to get out of bed in the morning and no longer wants to play with her grandchildren. She complains of decreased energy, increased sleep, and endorses feeling sad for many hours in a day for the past three weeks or so. Daughter notes that patient has been sleeping more and eating less and has been eating only one meal a day. Patient said she does not have the interest any more to do things she previously enjoyed. She denies any suicidal or homicidal thoughts and denies having any hallucinations. There has been no recent significant stressor that may have precipitated these symptoms

The correct answer is: A This patient appears to be suffering from a major depressive episode based on SIGECAPS criteria, with decreased energy, decreased interest, increased psychomotor retardation, increased sleep, depressed mood, and decreased appetite. Citalopram is a selective serotonin reuptake inhibitor (SSRI). SSRIs are the first line medications used to treat depression, including late onset depression. Studies have shown improvement in symptoms over a five week period compared to placebo. SSRIs are relatively safe and well tolerated. Common adverse effects include gastrointestinal upset, sexual dysfunction, and changes in energy. Citalopram, a SSRI, has shown to cause QT prolongation so it should not be used in any patient with a cardiac arrhythmia. Citalopram has also been shown to be beneficial in mood and neuropsychiatric symptoms in patients with moderate Alzheimer's disease.

A 69-year-old male complains of erectile dysfunction. History is pertinent for well controlled hypertension on propranolol and stable angina with sublingual nitroglycerin as needed. Physical examination reveals an overweight male with a smooth non-tender prostate, symmetrical testicular exam without tenderness or mass, intact sensation throughout the perineal area and normal pulses in the bilateral legs without cyanosis or edema. Structural examination reveals tissue T3-T7 to be neutral, sidebent right, rotated left. Which of the following is the most appropriate initial management? A. modify antihypertensive medication B. recommend vacuum assist device C.sleep study D. trial of sildenafil E. urology referral

The correct answer is: A Erectile dysfunction (ED) as a side effect of medication occurs in up to 25% of cases of ED. Common medications include SSRIs, antihypertensive medications such as beta-blocker and calcium channel blockers, alpha blockers such as finasteride, tricyclic antidepressants, digoxin and antihistamines. Changes in lifestyle and current medications should be tried before initiation of a PDE-5 inhibitor. Answer B: Vacuum assist devices are considered second line therapy after failure of lifestyle modification, medication changes and trial of phosphodiesterase type 5 inhibitors. Vacuum assist devices can be difficult to use and have a high drop-out rate. Answer C: Sleep apnea can contribute to cardiovascular damage due to frequent hypoxic episodes, however, there is little indication for a sleep study in this patient. In men with ED and sleep apnea, treatment with CPAP improved EF in some, but not all, studies. Answer D: Sildenafil is a phosphodiesterase type 5 inhibitors and is a first line medication in the treatment of erectile dysfunction after lifestyle and medication modifications. PDE-5 inhibitors are contraindicated in patients taking nitrates, as the combination of medication will precipitate dangerously low drop in blood pressure. Answer E: Initial evaluation of erectile dysfunction (ED) includes examination of risk factors, review of medications, physical examination and laboratory testing as indicated. Risk factors for ED include obesity, smoking, sedentary lifestyle, diabetes, stroke, vascular disease and depression. ED as a side effect of medication occurs in 25% of cases. After failure of lifestyle modification and medication changes, patient should be trialed on a phosphodiesterase type 5 inhibitors as a first line medication. Referral to urologist should be considered after failure of first line medication trial.

A 38-year-old woman presents to the physician with a complaint of fever and chills for 48 hours. She also complains of a cough with thick yellow sputum production and mild diffuse chest pain when coughing. She has no significant past medical history and does not take any medications. She smokes 1 pack of cigarettes per day. Her temperature is 38.3ºC (101ºF), pulse is 104/min, respirations are 24/min, and blood pressure is 144/88 mmHg. On physical examination, inspiratory crackles are noted in the right lower lobe. Her chest radiograph is shown in the exhibit. The decision is made to treat her as an outpatient. The most appropriate empiric pharmacologic treatment for this patient is A. doxycycline B. nafcillin C.oseltamivir D. oxacillin E. vancomycin

The correct answer is: A For patients with CAP, empiric antibiotic therapy is often required as the specific cause is not usually determined on initial evaluation. Outpatient therapy usually includes coverage against both typical and atypical organisms. In patients younger than 60 years of age, the most common organisms are S. pneumoniae, Mycoplasma, Chlamydia, and Legionella. Macrolides or doxycycline provide coverage against these organisms and are the first-line treatment. Respiratory quinolones such as moxifloxacin or levofloxacin or tigecycline are also effective monotherapy antibiotics. Penicillins and cephalosporins do not cover atypical organisms and should not be used as empiric antibiotic treatment in young, otherwise healthy patients because atypical organisms are likely. In patients over 60 years of age or those with significant comorbidities, typical CAP is more likely and second or third generation cephalosporins, amoxicillin/clavulanic acid, macrolides, and fluoroquinolones with pneumococcal coverage are appropriate. Answers B & D: Nafcillin and oxacillin provide coverage against methicillin-sensitive Staphylococcus aureus. They do not provide coverage against organisms causing typical CAP. Answer C: Oseltamivir is used for the treatment of infections caused by influenza A and B. It is not useful in treating bacterial infections. Answer E: Vancomycin is used for infections caused by methicillin-resistant S. aureus. It should not be used as empiric therapy for CAP.

A 55-year-old male ingested sildenafil and now has a sustained priapism. The cause of this phenomenon involves A. erection of the corpus cavernosum B. erection of the corpus spongiosum C. erection of the glans penis D. prolonged compression of perineal region E.vasospasm of the inferior pudendal artery

The correct answer is: A The corpus cavernosum is lined by vascular epithelium with blood flow supplied by the internal pudendal artery. Stimulation by the parasympathetic nervous system releases nitric oxide (NO) which stimulates cGMP, causing relaxation of the intracavernous smooth muscle and increased blood flow. Sildenafil, a phosphodiesterase type 5 (PDE-5) inhibitor, inhibits the enzyme that breaks down cGMP, thus increasing the interval of vasodilation to the corpus cavernosum. Unwanted erection that last more than 4 hours can causes ischemic (high flow) or non-ischemic (low flow) priapism, which left untreated can cause tissue necrosis. Priapism is considered an emergency condition and warrants immediate intervention. Answer B: The corpus spongiosum surrounds the urethra distally and forms the glans of the penis. The corpus spongiosum is not directly involved in priapism. Answer C: The glans of the penis is the distal extension of the corpus spongiosum, which is not engorged during erection. Answer D: Engorgement of the corpus cavernosum increases penile pressure secondary to increased blood flow, which compresses the post cavernous venules and obstructs venous drainage. Prolonged compression of the perineal area will compress the internal iliac artery and decrease blood flow to the inferior pudendal artery, but will no effect the venules the drain the corpus cavernosum. Answer E: During erection, increased nitric oxide causes smooth muscle relaxation in the corpus cavernosum vasculature, not vasoconstriction.

A 68-year-old male smoker complains of left leg burning, tingling, and numbness that is worse in the morning. He reports that he used to be able to walk around the block, but now must sit down and rest after walking just a few hundred feet because of cramping in his lower calf muscle. The cramping resolves soon after sitting down to rest. Physical examination reveals the left leg to be more pale and cool to touch in comparison to the right leg. The posterior tibial pulse is 1+ on the left and 2+ on the right. There are brawny brown deposits in the skin over the anterior shin bilaterally with associated trace edema. The most likely cause of his pain is A. arterial occulsion B. deep venous thrombosis C. peripheral neuropathy D.thrombophlebitis E. venous incompetence

The correct answer is: A The most likely cause is an arterial occlusion in this 68-year-old male smoker. The biggest risk factor for peripheral arterial disease is smoking. Other risk factors include diabetes mellitus, known atherosclerotic disease in the carotid, coronary, or renal arteries, and some rheumatologic conditions such as systemic lupus erythematous. He presents with complaints of intermittent leg claudication (pain in the calf with walking that resolves with rest), a pale, cool extremity, and decreased posterior tibial pulse. These are prominent history and physical findings in patients with peripheral arterial occlusive disease. The next step in management is to obtain ankle-brachial index (ABI) and consult a vascular surgeon if the ABI is < 0.9. The ABI is a ratio of the systolic blood pressure in the ankle compared to the arm. If the posterior tibial pulse would not have been palpable the next most appropriate test would be to attempt to find a Doppler signal for the pulse. If no Doppler signal was able to be found, the patient should be emergently seen by a vascular surgeon. The initial management of peripheral arterial occlusive disease with palpable pulses is to modify risk factors by recommending tobacco cessation, encouraging regular exercise to promote collateral flow, treatment of diabetes mellitus to goal hemoglobin A1C < 7%, and initiating statin and anti-platelet therapy. Answer B: A deep venous thrombosis (DVT) would present with calf pain, swelling, and tenderness to palpation. Exam findings would be swelling (> 3 cm difference in calf circumference), palpable cords, positive Homan's sign (tenderness in the calf with dorsiflexion of the foot) and venous distension of the superficial leg veins. This patient presented with pain, but did not have tenderness to palpation or swelling. The history and physical findings of claudication symptoms, decreased pulse, and cool, pale extremity make this more likely an arterial occlusion than a venous occlusion(thrombosis). If your suspicion is high for venous thrombosis you should obtain a duplex ultrasound of the lower extremity to evaluate for venous thrombosis. If it is positive for clot, the patient should be anti-coagulated with warfarin or one of the newer factor Xa inhibitors such as ribaroxaban, or fondaparinux. If the patient is treated with warfarin the patient will also need to be treated with heparin bridging until the INR is between 2-3. Answer C: Peripheral neuropathy can present with burning, tingling, numbness (paresthesias, dysesthesias) in the feet. It is characterized by progressive loss of large and small myelinated and unmyelinated nerve fibers in the lower extremities. Loss of vibratory sense and altered proprioception are the result of large fiber loss. Impairment of pain, temperature, and sensation is due to loss of small fibers. Loss of sensation typically begins in the distal feet and moves proximally into the calves at which time it can also be noted in the hands. This is described as a loss of sensation in the typical "stocking-glove" distribution. It does not present with claudication symptoms or changes in the distal pulse. Physical exam is positive for decreased sensation to mono-filament and may begin to develop decreased or loss of deep tendon reflexes. It is treated by controlling the patient's blood sugar and prevention of complications. The pain may be treated with tricyclic antidepressants such as amitriptyline, serotonin norepinephrine reuptake inhibitors such as duloxetine or venalfaxine, or anticovulsants such as gabapentin or pregabalin. Answer D: Thrombophlebitis is the inflammation in the vein and surrounding skin that occurs when a thrombus occurs in a superficial vein close to the skin. It is characterized by tenderness to palpation, erythema, and often warmth and swelling in the area of inflammation. It is commonly seen in the lower extremities. It is treated with warm compresses, elevation of the extremity, compression stockings, and analgesics/anti-inflammatory medications. Answer E: Chronic venous insufficiency can present with lower leg aching pain and cramping. It can also present with bilateral lower extremity swelling, varicose veins, and hemosiderin staining (brawny brown deposits) of the anterior shins. The swelling is usually worse in the evening or after prolonged standing and improves with elevation and first thing in the morning. This patient did have signs that are consistent with chronic venous insufficiency (hemosiderin staining and trace bilateral edema), but these findings were not likely to be causing the symptoms described. The exam findings that indicate that this is not due to chronic venous insufficiency are the claudication symptoms, pale/cool extremity and the changes in the palpable distal pulse. If it had been more consistent with chronic venous insufficiency the initial treatment would be compression stockings, elevation, and ambulation.

A 3-year-old male presents to the emergency department with chief complaint of drooling and that the patient is "just not acting right" per the mother. According to his mother, he was in his usual state of health until an hour prior to presentation, when she found him playing with her purse. Vital signs reveal a heart rate of 90/min, respirations at 22/min, and an oxygen saturation of 99% on room air. A plain film anteroposterior chest radiograph is obtained demonstrating the face of a coin in the upper chest. The object is most likely located within the A. esophagus B. left mainstem bronchus C.right mainstem bronchus D. right middle lobe E. trachea

The correct answer is: A The patient in this scenario likely swallowed a coin due to the fact that he was seen playing with his mom's purse prior to his symptoms starting and due to the shape of the object on x-ray imaging. The x-ray here shows the coin "en face" (facing you) which likely indicates that the coin is located in the esophagus rather than in the trachea. Coins located in the esophagus will be seen in the coronal plane (facing you on a PA or AP film) and coins seen in the trachea will be seen en face in the sagittal plane (facing you on a lateral film). Symptoms of a esophageal foreign body include retrosternal pain, retching or vomiting, coughing or gagging, and difficulty swallowing solids and liquids. Treatment of an esophageal foreign body depends on the object. Sharp objects need to be removed with endoscopy to prevent perforation. Food boluses may be removed first pharmacologically with nitroglycerin, glucagon, or nifedipine in adults. However, most foreign bodies fail pharmacologic management and require endoscopic removal to prevent esophageal perforation or stricture formation. Patients with small and smooth objects that of past the esophagus may be discharged and followed up with serial imaging and monitoring of the stool. Answer B: Anatomically, the right mainstem bronchus is wider, shorter, and more vertical than the left mainstem bronchus, thus making it more likely to be a site of foreign body aspiration. The trachea branches into the right and left mainstem bronchi at the fifth intercostal space. The location of branching is also known as the carina, which is important to identify on a chest x-ray of an intubated patient to ensure adequate placement of the endotracheal tube. Another anatomical difference is that the right bronchus branches into three lobar bronchi, while the left branches into two lobar bronchi. Answer C: The most common site of foreign body aspiration is the right mainstem bronchus, as this has a wider, shorter, and more vertical anatomical position compared to the left. Any child with respiratory distress should be evaluated for foreign body aspiration, with stabilization of ABCs always being the priority. The patient in this example is not in respiratory distress and more time can be taken for evaluation (i.e., obtaining a chest x-ray and neck x-ray). Answer D: The right middle lobe is often a common site for aspiration pneumonia in patients of all ages. This patient clearly has a foreign object on his radiograph, with no fever or infiltrates to suggest an infection. The chance of an object as large as a coin traveling as far down as the middle lobe is unlikely, as this is the smallest lobe and would involve passing from through very narrow bronchioles. Answer E: Patients who present with foreign objects in their trachea are typically emergent cases, as this implies that the entire airway distal to this point, including both mainstem bronchi, has been compromised. Initially, direct laryngoscopy can be attempted to directly visualize the object and attempt removal of the object. If one is unable to visualize the object and remove it further intervention would be required including obtaining an emergency airway distal to the site of obstruction until the object can be removed. Since this patient is crying during his evaluation, has normal vitals and no respiratory distress, it implies that his airway is patent and thus is not emergent.

A 56-year-old female is brought to the office by her daughter with pain in her fingers of her right hand. She is mildly confused and is unsure exactly how long she has had this pain. Her daughter says that her mother has complained about this pain for about a month and has been progressively more confused over the last week. She also says that her mother has been complaining of increased nausea for the last few months. A plain film radiograph of the right hand displays signs of osteopenia with multiple cysts described by the radiologist as brown tumors. The most likely etiology is A. hyperparathyroidism B.hypocalcemia C. multiple myeloma D. osteoporosis E. osteosarcoma

The correct answer is: A This patient presents with bone pain, nausea, and confusion, which can be seen together in hypercalcemia secondary to hyperparathyroidism. To remember the different features of hypercalcemia, think "bones, stones, abdominal groans, and psychiatric overtones." In severe hyperparathyroidism, patients may develop osteitis fibrosa cystica. "Brown tumors" may form in the bones of patients causing pain and leading to increased risk of fracture. These lesions are cysts filled with blood, fibrous tissue, and osteoclasts. Answer B: Patients with hypocalcemia can present with a variety of symptoms depending on both the acuity and degree of the hypocalcemia. Symptoms include muscle cramps or spasms, perioral or finger tingling, wheezing, and seizures. It would not cause the cystic lesions seen in this patient. Answer C: Multiple myeloma is a proliferative disorder of plasma cells, leading to a monoclonal gammopathy. Infiltration of the bone marrow with plasma cells causes impaired erythropoiesis and anemia. It also causes lytic lesions in the bones, leading to bone pain and pathologic fractures. Hypercalcemia also occurs as a result of these processes, and patients may present similarly with confusion. The cystic bone lesions seen on her x-ray would not be expected in patients with this condition. Answer D: This patient's age (> 50 years) and sex (female) put her at increased risk for osteoporosis, which is a condition of low bone density. Patients are at high risk for fractures, particularly of the hips and the vertebrae, but cystic bone lesions are not associated with this condition. Additionally, this diagnosis would not explain her confusion as osteoporosis is not associated with hypercalcemia. Answer E: Osteosarcoma is an uncommon primary bone malignancy that is most frequently seen in children. In adults, it is typically the result of Paget's disease. These are most frequently found at the end of long bones, such as the tibia or humerus, and appear as a "sun burst" pattern on x-ray.

A 30-year-old G2P2 female presents to her obstetrician complaining of vaginal pain. She recently gave birth to a full-term, 4.2-kg (9.3-lb) boy 7 days prior via a spontaneous vaginal delivery. Her labor was complicated by shoulder dystocia, which required a mediolateral episiotomy to aid fetal expulsion. Since her hospital discharge, she has been breastfeeding without complications and denies any abdominal pain or vaginally bleeding. Her temperature is currently 37.8°C (100.2°F). Gynecological examination reveals a nontender, firm uterus with the fundus approximately midway between the symphysis pubis and umbilicus. The episiotomy site is moderately erythematous with swelling around the periphery. The sutures are unevenly spaced and thick, white fluid is draining from the laceration. Which of the following is the most appropriate therapy for this patient? A. irrigation and debridement B. IV clindamycin and gentamicin C. oral metronidazole D.sitz baths E. suture replacement

The correct answer is: A This patient's vaginal pain is due to a superficial infection at her episiotomy site. An episiotomy is a surgical incision performed by the obstetrician within the perineum and posterior vaginal wall. Indications for episiotomy include a need to create a larger pelvic outlet (e.g., fetal macrosomia), high likelihood of a serious perineal tear during labor, instrumental deliveries (e.g., forceps), and shoulder dystocia. The two most common types are median (incision is placed midline between vagina and anus) and mediolateral (45° lateral to midline on either right or left). In the past, routine episiotomies were frequently performed because they were thought to decrease perineal trauma and minimize postpartum pelvic floor dysfunction. However, more recent studies have found that episiotomies can actually cause worse perineal damage than natural tears and are associated with significant postpartum complications, including bleeding, infection, dehiscence, extension with damage to the anal sphincter, dyspareunia, and rectovaginal fistula formation. Episiotomy infections commonly present 6-8 days after labor and are more commonly seen with a longer second stage of labor, operative vaginal delivery, mediolateral episiotomy, third and fourth degree lacerations and the presence of meconium stained amniotic fluid. The majority of infections remain localized to the superficial tissues and present with mild to moderate swelling, warmth, erythema and purulent discharge. However, more serious infections, including cellulitis, perineal abscess, and, rarely, necrotizing fasciitis, can develop. It is necessary to open the incision when there is evidence of breakdown of the primary perineal laceration repair (indicated by uneven sutures). Appropriate management includes removal of all suture material; and thorough irrigation and debridement of infected tissue under local, regional, or general anesthesia. Broad spectrum antibiotic therapy may be considered, but usually is not necessary unless there are signs of cellulitis or systemic infection present. Answer B: Intravenous clindamycin and gentamicin are used to treat endometritis, the most commonly seen postpartum infection. This patient is afebrile and has a nontender uterus, making the diagnosis of postpartum endometritis unlikely. Her presentation is consistent with episiotomy site infection. Antibiotic therapy is usually not indicated for episiotomy infections that are localized to the superficial tissue surrounding the incision site. If there were signs and symptoms suggestive of cellulitis or systemic infection (e.g., expanding erythema, fever, chills) it would be appropriate to initiate oral antibiotic therapy. Answer C: Metronidazole is an antimicrobial used to treat infections caused by Giardia, Entamoeba, Trichomonas, Gardnerella vaginalis, H. pylori and anaerobes. In gynecology, it is most commonly used to treat bacterial vaginosis (due to Gardnerella vaginalis) and sexually transmitted diseases caused by Trichomonas. This patient is not presenting with signs and symptoms suggestive for either of these infections (e.g., vaginal discharge, foul odor); therefore, oral metronidazole would not be appropriate. Of note, the use of metronidazole in breastfeeding women is controversial. Animal studies have shown potential for tumorigenicity. The American Academy of Pediatrics currently recommends that women taking high doses of metronidazole postpone breastfeeding and discard milk while taking the medication. Answer D: Sitz baths are commonly used, along with NSAIDs and cold compresses, to relieve the pain associated with an episiotomy. This patient's examination indicates an episiotomy infection, which requires more aggressive treatment with irrigation and debridement. Answer E: Suture replacement alone would not be appropriate for this patient. Episiotomy infections require removal of sutures for adequate irrigation and debridement. The decision to replace the sutures after treatment depends on the size of the affected area. Small defects can be left alone to heal by granulation. Larger areas are usually resutured once adequate granulation tissue has formed on the wound surface.

An elderly patient presents to the office for a routine wellness examination. Although a history is unremarkable, physical examination reveals a pulsatile abdominal mass with ecchymoses around the umbilicus. Which of the following has the highest risk for this patient's most likely condition? A. 62-year-old male with medically-managed hypertension B. 66-year-old male with a 5 pack-year smoking history C. 67-year-old alcoholic female D. 77-year-old male with uncontrolled diabetes mellitus type II E.78-year-old female with hypertriglyceridemia

The correct answer is: B Abdominal aortic aneurysm (AAA) is an abnormal localized dilation of the aorta. Most AAA's occur between the renal arteries and iliac bifurcation. The incidence increases with age. AAA's are rare before the age of 50 - the average age at time of diagnosis is 65- to 70-years of age. They are much more common in men, and are typically caused by atherosclerotic weakening of the aortic wall among other predisposing factors such as trauma, hypertension, vasculitis, smoking, and family history. Syphilis and connective tissue abnormalities like Marfan's disease are associated with thoracic aneurysms, but they may involve the lower aorta as well. AAA's are typically asymptomatic and found incidentally on imaging. They may present simply with a sensation of "fullness" without or without pain. If present, pain will be located in the hypogastrium and lower back and will typically be described as throbbing. A pulsatile mass can be felt on abdominal exam, as in this patient. The sudden onset of severe pain in the back or lower abdomen, radiating to the groin, buttocks, or legs suggests expansion and impending rupture. Grey Turn's sign is characterized by ecchymoses on the back and flanks and Cullen's sign is ecchymoses around the umbilicus (as seen in this patient); both are signs of impending rupture as well. The triad of abdominal pain, hypotension, and palpation pulsatile abdominal mass indicates a ruptured AAA requiring emergent laparotomy. Patients may also present as hemodynamically unstable, with syncope, or they may simply present with nausea or vomiting. The likelihood that an aneurysm will rupture is increased for those with aneurysm diameter greater than 5.5 cm, a faster rate of expansion (>0.5 cm over a six-month period), those who continue to smoke, and in females. According to the USPSTF, a one-time screening for AAA is recommended for men ages 65 to 75 who have smoked. There are further recommendations for this screening in men ages 65 to 75 who have never smoked but who have a first-degree relative who required repair of an AAA or died from a ruptured AAA. Answer A: This patient's age does not fall within the range for recommended screening. There is no information provided about his smoking history. Further, though hypertension is a predisposing factor that allows for atherosclerotic weakening of the aortic wall, the presence of hypertension is not included in the screening recommendations. Also, his hypertension is medically-managed, so shearing forces caused by elevated pressures on this patient's blood vessels are lessened in this patient. Answer C: There are recommendations against screening for AAA in women. Further, alcoholism is not a risk factor for AAA. Answer D: Men who are over age 75 are unlikely to benefit from screening. Uncontrolled type II diabetes mellitus is not an indication for screening. Answer E: Patients over 75-years-old are unlikely to benefit from screening. Further, this patient is a female and she is precluded from screening. Finally, hypertriglyceridemia is not included in the recommendations for screening.

A 10-year-old female presents to the office with chronic cough of three months' duration that awakens her twice weekly. Review of systems is negative for recent illness, chest congestion, nasal discharge, upset stomach, and ear pain. Pulse oximetry reveals an oxygen saturation of 99% on room air. Physical examination reveals pink and moist oral and nasal mucosa without lesions or exudation, a neck that is absent for lymphadenopathy, and normal heart sounds. Auscultation of the chest reveals diffuse mild expiratory wheezes. Spirometry performed during the visit reveals an FEV1 that is 72% of predicted values. The most appropriate pharmacologic therapy for long-term maintenance is A. albuterol as needed B. daily fluticasone/salmeterol C.daily montelukast D. daily salmeterol E. prednisone dose pack

The correct answer is: B Asthma is defined by a triad that includes airway inflammation, airway hyperresponsiveness, and reversible airflow obstruction. It can occur at any age and can have many triggers, including pollen, dust, mold, animals, cold air, viral infections, tobacco smoke, medications, and exercise. The above triad manifests clinically as dyspnea, wheeze, chest tightness, and cough. Severity varies, though symptoms typically begin within 30 minutes of irritant exposure. Symptoms are usually worse at night and wheeze can be on both inspiration and expiration, and is the most common finding on physical examination. Though the most common cause of wheeze is asthma, many conditions mimic large-airway bronchospasm that can cause wheeze, including congestive heart failure, chronic obstructive pulmonary disease, cardiomyopathy, and lung cancer. Pulmonary function tests are required for diagnosis and spirometry before and after bronchodilators can confirm diagnosis by demonstrating reversible airway obstruction. Asthma severity is determined by considering the following factors: reported symptoms over the previous two to four weeks, current level of lung function, and number of exacerbations requiring oral glucocorticoids per year. Various charts are available that define categories of asthma severity, typically using frequency of daytime asthma symptoms per week and number of nocturnal awakenings per month. In this patient, who is awakened up to twice weekly, moderate persistent asthma is likely. These categories become important in determining appropriate chronic treatment options. This patient has moderate persistent asthma, which is characterized by daily asthma symptoms, nocturnal awakenings more than once weekly (though not nightly), daily need for SABA (short-acting beta agonist), FEV1 between 60 and 80% of predicted, FEV1/FVC 1-5% below normal range, or two or more exacerbations requiring oral glucocorticoids yearly. This question does not provide us much information beyond the FEV, so we must use this to diagnose. Inhaled glucocorticoids are recommended as the initial and primary therapy in all patients with moderate persistent asthma. Long-term control should be through medium-dose inhaled glucocorticoids alone or the combination of low-dose inhaled glucocorticoids plus a long-acting beta agonist (LABA). Combination glucocorticoid/LABA inhalers include budesonide-formoterol, mometasone-formoterol, fluticasone-salmeterol. One could even consider a medium-dose inhaled glucocorticoid, though this is more appropriate for patients whose symptoms have been stable, though not well-controlled in the preceding weeks. Combination LABA/glucocorticoids are typically more effective than higher doses of inhaled glucocorticoids (IGC). Think of the basics of asthma treatment in pyramid form like below: Intermittent: SABA Mild Persistent: SABA + low-dose IGC Moderate Persistent: SABA + low-dose IGC + LABA Severe Persistent: SABA + high-dose IGC + LABA

A 35-year-old female with past medical history of unspecified anemia presents with mild generalized fatigue and some occasional instances of self-limited diarrhea. Physical examination reveals normoactive bowel sounds without abdominal tenderness and hepatosplenomegaly. Laboratory studies are obtained and reveal the following: Hemoglobin 11.2 g/dL Mean corpuscular volume 115 mcm3 Alkaline phosphatase 205 U/L Alanine aminotransferase 58 U/L Aspartate aminotransferase 89 U/L IgA endomysial antibody positive The most likely diagnosis is A. alcoholic hepatitis B. celiac disease C. Crohn's disease D. tropical sprue E. Whipple's disease

The correct answer is: B Celiac disease is a small bowel disorder characterized by mucosal inflammation, crypt hyperplasia, and villous atrophy. It typically occurs upon exposure to dietary gluten and improves with withdrawal of the substance. Serologic testing for celiac disease and the common use of endoscopy has complicated the definition, since these tests have identified patients who appear to have the disease but have variable degrees of histopathologic changes and/or symptoms. Testing for celiac disease should be considered in patients with gastrointestinal symptoms including chronic or recurrent diarrhea, malabsorption, weight loss, and abdominal distension, or bloating, including patients with symptoms suggestive of irritable bowel syndrome or severe lactose intolerance. Also, patients without other explanations for iron deficiency anemia, folate, or vitamin B12 deficiency, persistent elevation in serum aminotransferases should be considered. This patient certainly manifests the macroscopic anemia (likely secondary to B12 and folate malabsorption) and elevated aminotransferases. All testing should be performed while patients are on a gluten-rich diet. No single test can confidently establish the diagnosis of celiac disease in every individual. Thus it is most important to recognize the many clinical features that can be associated with the disease. Testing should begin with serologic evaluation. Immunoglobulin A (IgA) anti tissue transglutaminase (TTG) antibody is the single preferred test for detection of celiac disease in individuals over the age of two years. Patients with positive serology, and those with high probability of disease, regardless of the serology, should undergo an upper endoscopy with small bowel biopsy to confirm the diagnosis. This might demonstrate increased intraepithelial lymphocytes or flat mucosa with total mucosal atrophy, complete loss of villi, and crypt hyperplasia. A variety of serologic studies have been described to aid in the diagnosis of celiac disease, including IgA endomysial antibody (IgA EMA), IgA/IgG tissue transglutaminase antibody (IgA/IgG tTG), and IgA/IgG deamidated gliadin peptide (IgA/IgG DGP). Serum IgA EMA and tTG have the highest diagnostic accuracy, while the DGP tests have lower diagnostic accuracy and frequent false positive results and are therefore no longer recommended for initial diagnostic evaluation or screening. IgA EMA testing is moderate sensitive and highly specific for untreated celiac disease. Serum levels of IgA EMA fall on a gluten-free diet and the test often becomes negative in treated patients. This presents with some very non-specific symptoms. It is important to note that patients may present with classic signs, including diarrhea with bulk-foul-smelling, floating stools due to steatorrhea and flatulence. Malabsorption may cause growth failure, weight loss, anemia, neurologic disorders from B vitamin deficiencies, and osteopenia from calcium and vitamin D deficiencies. The patient's mild fatigue may be a manifestation of her megaloblastic anemia. Her stools are notable (though non-specific), though most patients today do not present with classic celiac disease but with atypical symptoms or an asymptomatic presentation. Most present with subclinical disease that can exist in a very mild form and may go largely undetected because of the unspecific symptoms.

A 35-year-old female presents to the office for a routine health maintenance examination with concerns about colon cancer because her paternal grandmother died of colon cancer and her 55-year-old mother was recently diagnosed with colorectal cancer. History reveals that she is healthy and has no medical problems. She does not smoke or drink alcohol. Physical examination and routine laboratory studies are normal. The most appropriate recommendation for screening in this patient includes A. annual fecal occult blood test starting this visit B. colonoscopy every 5 years starting at the age of 40 C. elective colectomy D.genetic testing E. sigmoidoscopy every 5 years starting at age 45

The correct answer is: B The United States Preventative Services Task Force recommends at all individuals between the ages of 50 and 75 who are at average risk for colorectal cancer undergo routine screening using high-sensitivity fecal occult blood test (FOBT), flexible sigmoidoscopy, or colonoscopy. Colonoscopy is the preferred method by most physicians today because it allows for complete visualization of the colon and any polyps or suspicious lesions can be removed and biopsied immediately, providing critical diagnostic information efficiently. Patients with certain risk factors may require earlier and more frequent surveillance. Patients who are considered at increased risk include those with a first-degree relative with colon cancer or advanced adenoma diagnosed at age < 60 or two first-degree relatives diagnosed at any age. These patients should be advised to have screening colonoscopy every 5 years starting age 40 or 10 years younger than the earliest diagnosis in their family, whichever comes first. If the first-degree relative was diagnosed at age >60 then the patient should have screening colonoscopy every 10 years starting at 40 years old. This patient has one first-degree relative diagnosed with colorectal cancer at < 60 years of age. Therefore, it is recommended that she undergo routine screening with colonoscopy every 5 years. Answer A: Annual fecal occult blood tests (FOBT) starting at the age of 50 is an alternative method approved for colorectal cancer screening in patients who are average-risk (ie. have no significant risk factors). Studies have shown that yearly FOBT testing in patients aged 50-80 can reduce the number of colorectal cancer deaths by 15-33%. A positive FOBT study requires a more thorough evaluation and patients usually end up undergoing a colonoscopy. Therefore, its use is not recommended in patient's with risk factors for colorectal cancer because it may delay diagnosis and early intervention. Answer C: Elective colectomy is offered as prophylactic therapy for certain patients who are at significantly increased risk of colorectal cancer. These include patients who have had ulcerative colitis for > 10 years or those with familial adenomatous polyposis. This patient's situation does not warrant such treatment. Answer D: Hereditary gene mutations can significantly increase one's risk for developing colorectal adenomas and adenocarcinoma, as well as other cancers, at a young age. The two most common are familial adenomatous polyposis (FAP) caused by a mutation of the APC gene and hereditary nonpolyposis colorectal cancer (HNPCC) caused by a DNA mismatch repair gene mutation. Genetic testing for hereditary forms of colorectal cancer may be offered to certain patients who have multiple first-degree family members with colorectal cancer, first-degree relative with FAP or HNPCC, present with multiple polyps at a young age, or who present with other types of cancer that are often associated with these syndromes ( Patients with HNPCC are also at increased risk for cancers of the stomach, small intestine, liver, gallbladder ducts, upper urinary tract, brain, and skin). This patient does not require genetic testing. Answer E: Sigmoidoscopy every 5 years starting at age 50 is an alternative option for routine colorectal cancer screening in patients who are at average-risk. This patient is at increased risk because she has a first-degree relative who was diagnosed before the age of 60. Therefore, she requires screening at the younger age of 40, preferably with a colonoscopy

For each numbered item (patient presentation), select the one heading (treatment) most closely associated with it. Each lettered heading may be selected once, more than once, or not at all A. citalopram B. donepezil C. memantine D. omega-3 fatty acids E. selegiline A 79-year-old disheveled male in the company of his nephew presents to his primary care physician with decreased appetite and worsening forgetfulness. He is recently widowed, and now resides alone. He does visit the YMCA on a daily basis which he enjoys. The patient is oriented to place and self, but not to date and time. Mini-mental status examination is performed and his score falls in the range of mild dementia, and he is able to draw a clock on the clock drawing test.

The correct answer is: B The patient's MMSE reveals mild dementia and there is not enough information to diagnose depression, as he still has interest in his daily activity of going to YMCA, and does not seem to endorse a depressed mood. Donepezil is a cholinesterase inhibitor used to treat mild to moderate symptomatic dementia. Decreased production of choline acetyl transferase leads to a decrease in the synthesis of acetylcholine. Treatment with cholinesterase inhibitors like Donepezil have shown to improve cognition and activities of daily living. Donepezil is preferred because it is safe and well tolerated. Cholinergic side effects are transient and mild. Bradycardia is the most common side effect. Also, the drug is highly effective due to decreased peripheral catabolism. Other cholinesterase inhibitors used to treat the disease include tacrine, rivastigmine and galantine. Tacrine is rarely used due to hepatotoxicity. Cholinesterase inhibitors can also treat the neuropsychiatric symptoms. Although cholinesterase inhibitors have proven efficacy in mild to moderate dementia, they have not shown to be disease modifying.

A 43-year-old male complains of pain and swelling of his right knee that started 12 hours ago. History reveals the patient is not sexually active. Past medical history is significant for hypertension controlled by hydrochlorothiazide and amlodipine, diabetes controlled by metformin, and dyslipidemia treated with simvastatin. Physical examination reveals an erythematous right knee joint that is exquisitely tender to palpation. Range of motion testing is limited due to pain. You note paravertebral muscle spasms from T12-L2 with tenderness to palpation in this region. Laboratory studies reveal: Na+ 135 mEq/L K+ 3.5 mEq/L Cl- 98 mmol/L HCO3- 26 mEq/L BUN 20 mg/dL Cr 0.9 mg/dL Glucose 180 mg/dL The most appropriate recommendation to prevent recurrence is to A.discontinue amlodipine B. discontinue hydrochlorothiazide C. discontinue simvastatin D. prescribe glyburide E. prescribe indomethacin

The correct answer is: B This patient is suffering from an acute attack of gout. Gout is a form of arthritis usually affecting a single joint, most commonly the first metatarsophalangeal joint. Other common joints affected are the knees and ankles. It is caused by deposition of sodium urate crystals in the joints. This occurs when uric acid levels are high either from overproduction or from underexcretion. Symptoms include sudden onset of pain and swelling. To diagnose gout a joint aspiration should be done which will show needle shaped, negatively birefringent crystals. Treatment of an acute gout attack is NSAIDs, usually indomethacin. To prevent future gout attacks medicines such as allopurinol and probenecid are used as well as avoiding risk factors if possible. One risk factor for acute gout attacks is diuretic use. Hydrochlorothiazide is a thiazide diuretic which inhibits the distal convoluted tubule sodium and chloride resorption. It is a first line drug used in the treatment of hypertension. In this patient it is controlling his hypertension but it should be changed to another antihypertensive because of its side effects. Hydrochlorothiazide causes electrolyte abnormalities such as hyponatremia, hypokalemia, and hypochloremia. This patient has lab values at the lower limits of normal for sodium, potassium and chloride. It also causes hyperuricemia which is the likely cause of this patient's gout attack. Another antihypertensive drug should be selected based on the patient's health history. A beta blocker could work but caution should be taken because he is diabetic, it would also not be a good choice if the patient had obstructive lung disease or had bradycardia. An ACE-inhibitor could also be a good choice, they have serious side effects of angioedema and renal impairment but more commonly can cause a dry cough. As seen in this vignette, there are TART changes from T12-L2 that are likely the result of viscerosomatic reflexes from the lower extremity pathology. You can see this with any lower extremity dysfunction. Answer A: Amlodipine is a dihydropyridine calcium channel blocker used to treat hypertension. It's mechanism of action is to inhibit calcium ion influx into vascular smooth muscle. Common side effects include peripheral edema, fatigue, and palpitations. It does not affect uric acid levels. Answer C: Simvastatin is used to treat hypercholesterolemia in patient's who fail lifestyle modifications. Side effects of statin medications include myopathy, rhabdomyolysis, and hepatotoxicity. Liver function tests must be monitored and the medication discontinued if they are elevated or if the patient develops muscle aches. Answer D: This patient is presenting with an acute gout attack. His glucose level is within normal limits for a random glucose level. There is no need to adjust diabetic medications at this time. Metformin has side effects including lactic acidosis, diarrhea, nausea, vomiting, and indigestion. It does not affect the uric acid level or precipitate gout attacks. Glyburide is in the sulfonylurea drug class indicated for treatment of diabetes mellitus type 2. Answer E: Indomethacin is used to treat acute gout attacks. It is a non-steroidal anti-inflammatory drug. It does not prevent future gout attacks. Medications used to prevent future attacks include probenecid and xanthine oxidase inhibitors like allopurinol and febuxostat.

A 56-year-old female complains of palpitations and an unintentional 9 kg (20 lb) weight loss in the past six months. History reveals that her menstrual periods stopped approximately 7 years ago. Vital signs include a blood pressure of 135/85 mmHg and a heart rate of 110/min. Physical examination reveals a non-tender enlarged thyroid gland without nodules. Structural examination reveals tissue texture changes at T2 associated with an upper thoracic flexion hump. Laboratory studies reveal the following: TSH 0.05 mU/L free T3 380 mmol/L free T4 22 mmol/L A thyroid stimulating immunoglobulin test is positive. The most appropriate initial management is A. levothyroxine B. methimazole C. propylthiouracil D. radioactive iodine ablation E. thyroidectomy

The correct answer is: B This patient's clinical picture points to a diagnosis of hyperthyroidism. The most common cause of hyperthyroidism in the United States is Grave's disease. Normally a homeostasis is maintained between the thyroid gland, the hypothalamus and the anterior pituitary gland. The thyroid glands follicular cells produce T3 and T4, which is regulated via a feedback loop through the anterior pituitary's production of TSH. The levels of Thyroid stimulating hormone (TSH) is in-turn regulated by hypothalamic production of Thyrotropin releasing hormone (TRH). The underlying etiology in Graves's disease is an autoimmune phenomenon - due to thyroid stimulating immunoglobulins, which bind to and activate the thyrotropin receptors, which leads to increased secretions of thyroid hormones from the thyroid gland. Graves disease is a constellation of thyrotoxicosis, an enlarged symmetrical goiter, dermopathy and ophthalmopathy. Clinical laboratory values will illustrate a markedly decreased or undetectable TSH and an elevated T3 and / or T4. Clinical symptoms of hyperthyroidism include those of increased metabolic activity, including: fatigue and generalized weakness, weight loss despite an increased appetite, heat intolerance, diaphoresis, restlessness, tremors, insomnia, palpitations, tachycardia, atrial fibrillation, increased frequency of bowel movements, eye complains including diplopia and blurred vision, hair loss, pretibial swelling, decreased libido. The goal of treatment with graves disease is to decrease thyroid synthesis and ameliorate symptoms of hyperthyroidism. There are advantages and disadvantages to different treatment modalities including therapy with Thionamides (Propylthiouracil and Methimazole), radioactive iodine ablation and surgical excision. Answer A: Levothyroxine would be an incorrect choice. Administering a synthetic thyroid hormone replacement to a patient with symptomatic hyperthyroidism would exacerbate her symptoms and would not be indicated in this scenario. Answer C: Thionamides including propylthiouracil (PTU) and methimazole are primary first line choices in the treatment of hyperthyroidism due to Graves disease. PTU is preferred as a treatment of hyperthyroidism in the first trimester of pregnancy. However, PTU is associated with more toxic side effects as compared to methimazole. Both PTU and methimazole can induce unwanted side effects including: pruritus, rash, urticaria, arthralgias, arthritis, fever, abnormal taste sensation, nausea, vomiting, agranulocytosis and hepatotoxicity. Due to increased association of PTU with hepatotoxicity which can lead to hepatocellular damage, liver failure and increased mortality, a recent meeting of the American Thyroid Association and USDA, it was recommended that PTU should not be prescribed as first line drug in children or adults. Answer D: Radioactive iodine ablation is a plausible treatment choice for Graves disease but would be not be a first line choice in this patient. The initial goal in the treatment of hyperthyroidism is achieving a euthyroid state. Radioactive iodine is administered as an oral formulation, is rapidly absorbed in the gastrointestinal tract, is concentrated in the thyroid gland and eventually leads to gland destruction and ablation of the thyroid gland within 2 - 3 months. It is preferable to pre-treat patients with a thionamide (propylthiouracil or methimazole) to ameliorate symptoms of hyperthyroidism before radioiodine treatment. The treating physician should be cognizant that radioiodine use is absolutely contraindicated during pregnancy as it can result in fetal hypothyroidism. Answer E: Thyroidectomy is incorrect and would not be part of the initial therapy for this patient. Surgery provides a rapid and essentially permanent cure of hyperthyroidism but in turn leads to permanent hypothyroidism and the need for lifelong supplementation with thyroid hormone. It is indicated in patients who have a very large and obstructive goiter or in pregnant women who are allergic to anti-thyroid medications. Thyroidectomy is not recommended as a first line therapy for Grave's disease.

A 26-year-old male presents to the urgent care clinic with right elbow swelling and pain. History reveals he was playing a game of softball recently and hit his elbow on the ground while sliding. Vitals are obtained and reveal 37ºC (98.7oF), a blood pressure of 126/74 mmHg, a heart rate of 84/min, and a respiratory rate of 18/min. Physical examination reveals focal swelling of the right olecranon process that is tender, warm, and erythematous over a well-healed posterior abrasion. Range of motion testing, muscle strength, circulation, and neurologic testing are symmetrical. The most appropriate management at this time is A. amoxicillin-clavulanate B. bursa aspiration C. corticosteroid injection D.piperacillin-tazobactam E. rest, ice, compression, elevation, and ibuprofen

The correct answer is: B This presentation is consistent with an infected olecranon bursitis. Given the superficial location of the olecranon bursa, it is a common site of irritation that can lead to inflammation of the bursa, and any small break in the skin can lead to subsequent cellulitis or infected fluid in the bursa. If there is symptomatic swelling, the fluid should first be aspirated using a large bore needle and the fluid should then be sent for cell count, culture, and crystal analysis to determine the etiology. Infectious bursitis should be treated with oral antibiotics if the patient is hemodynamically stable. If there is any evidence of sepsis, intravenous antibiotics may be necessary. The most common differential diagnoses includes infectious bursitis, gout, pseudogout, which can be determined from the fluid studies. Other common sites of bursitis include the ischium, greater trochanter, and pre-patellar regions.

A 76-year-old female presents to your office stating that she is "walking funny." She first noticed this abnormality eight months ago when her left hip began hurting. She had been diagnosed with mild to moderate degenerative joint disease of her left hip. Physical examination reveals her right hip to drop when standing on her left leg. This is most likely due to weakened A. knee flexors B. hip abductors C. hip adductors D. hip extensors E. hip flexors

The correct answer is: B Trendelenburg gait is seen in individuals with hip-joint problems as a compensation for muscle weakness or instability. It is caused by weak hip abductors during stance phase as which time the contralateral hip appear to tilt down. This gait pattern avoids the stress put on the hip abductors through the normal stance phase of gait by shifting the body weight over the center of the hip-joint, decreasing the moment arm and effectively decreasing the effort needed from the hip abductors. In an individual with a normal hip and gait, the hip-joint force is produced by the body weight times the moment arm, which is the distance from the center of gravity of the body to the hip-joint. This is normally balanced by the force exerted by the abductors. The gluteus medius is the primary abductor and it is innervated by the superior gluteal nerve (L4-S1). Answers A & E: An abnormality associated with the knee flexors is with excessive knee flexion (not weakness), which may attenuate limb advancement. This type of gait is referred to as a crouched gait and may present with a knee flexion contracture. Excessive hip flexion (not weakness) may also contribute to the knee flexion deformity. Furthermore, excessive hip flexion may also interfere with limb advancement in swing phase. Answer C: The hip adductors play only a minor role in gait. They are responsible for bringing the feet toward the line of progression during the swing phase, decreasing the energy demands of walking. The adductor group is composed of the Adductor brevis, longus, magnus, and minimus as well as the pectineus, gracilis and obturator externus. These muscles are innervated by the obturator nerve (L2-L4). Answer D: In individuals with weak hip extensors, they will demonstrate a Gluteus Lurch gait. The patient will present in a lordotic position while walking, which keeps the center of gravity behind the hip. The primary hip extensor is the gluteus maximus and the majority of the hamstring with the inferior gluteal nerve (L4-S1) and sciatic nerve (L4-S2) providing innervations, respectively.

A 52-year-old female presents to your office after a random blood glucose level of 250 mg/dL was discovered at a screening health fair. She is sent for some additional lab work and you are awaiting the results before initiating pharmacologic therapy. Which of the following would establish a diagnosis of diabetes mellitus? A. 2-hour blood glucose level of 190 mg/dL during a 75 gram oral glucose tolerance test B. fasting blood glucose of 130 mg/dL C. hemoglobin A1C of 6.0% D. random blood glucose of 150 mg/dL with classic symptoms of hyperglycemia E.random blood glucose of 240 mg/dL without symptoms

The correct answer is: B Type 2 diabetes mellitus occurs because of tissue resistance to insulin which leads to hyperglycemia. Eventually the β-islet cells of the pancreas are unable to produce sufficient insulin to meet the bodies demand. Usually type 2 diabetes mellitus is diagnosed later in life with gradual onset of symptoms of polyuria, polydipsia, weight loss, and vision changes. Untreated it can lead to retinopathy, nephropathy, neuropathy, and hyperosmolar hyperglycemic non-ketotic coma. There are several ways to diagnose diabetes according to the American Diabetes Association. These include hemoglobin A1C greater than or equal to 6.5%, fasting plasma glucose greater than or equal to 126 mg/dL, oral glucose tolerance test with 2 hour blood glucose level greater than or equal to 200 mg/dL, or random plasma glucose greater than 200 mg/dL in a patient with classic symptoms of diabetes. These tests should always be repeated for confirmation.

A healthcare worker is stuck with a needle that was known to be in contact with a HIV positive patient. What is the mechanism of action of the most appropriate prophylactic medication? A. inhibition of viral entry B. inhibition of viral replication C. increased cytotoxic T-cell count D. increased interleukin-2 levels E.inhibition of protease

The correct answer is: B Viral RNA incorporates into dsDNA by the viral reverse transcriptase enzyme. Nucleoside and Non-Nucleoside reverse transcriptase inhibitors act to block the action of the reverse transcriptase enzyme. Nucleoside Reverse Transcriptase Inhibitors (NRTIs) act by direct competition of viral reverse transcriptase by mimicking other nucelosides and are incorporated into the DNA strand and preventing the addition of natural nucleosides into the DNA strand. This halts production of new virions . NRTI medications include Abacavir, Tenofovir and Emtricitabine. Non-Nucleoside Reverse Transcriptase Inhibitors (NNRTIs) acts by direct binding and inhibition of the reverse transcriptase enzyme. NNTRIs include Delavirdine, Efavirenz and Nevirapine. A common regimen includes tenofovir-emtricitabine (NTRIs) combined with raltegravir (integrase inhibitor). Alternative regimens that are equally acceptable are tenofovir-emtricitabine combined with ritonavir-boosted atazanavir or ritonavir-boosted darunavir. Drugs that should NOT be used for post exposure prophylaxis (PEP) are abacavir and nevirapine, which may cause severe and sometimes life-threatening side effects. Answer A: Fusion inhibitors, such as Enfuvirtide, inhibits the entry of HIV into the CD4 cell by changing the structure of the T-20 cell membrane protein and preventing binding and fusion by the gp41 viral membrane protein. Fusion inhibitors are only active when injected subcutaneously. The inconvenient administration and development of other medications against resistant viral strains limits the utility of this medication. Answer C: Cytotoxic T-cells, also known as CD8 T-cells, destroy abnormal cells by recognizing and binding to major histocompatibility complex (MHC) class 1 antigens on the cell surface. IN HIV infections, reduced numbers of CD4 cells may lead to a decrease in the activation and survival of cytotoxic CD8 cells. CD8 cells can destroy viral infected cells, but do not prevent disease progression. Answer D: Interleukin-2 (IL-2) is necessary to the growth, proliferation and differentiation of T-cells during an immune response and is essential for the development of T-cell immunologic memory. IL-2 is manufactured using recombinant DNA and is sold as aldesleukin (Proleukin). Recent SILCAAT and ESPIRT trials demonstrated that IL-2 therapy provided substantial and sustained increase in the CD4+ cell count, as compared with antiretroviral therapy alone. But, despite the increase in CD4+ cell count with IL-2 and antiretroviral therapy, both trials yielded no clinical benefit in change of opportunistic disease or death in either study. Answer E: Tirpranavir and nelfinavir are protease inhibitors that are not recommended for the use of post-exposure prophylaxis for occupational workers. Other medications not recommended include didanosine (NRTI) and nevirapine. PEP with nevirapine has resulted in some severe adverse events including hepatotoxicity and Stevens-Johnson syndrome, thus, this drug is contraindicated for post-exposure prophylaxis.

A 71-year-old male presents for a routine wellness examination with difficulty breathing. History reveals his symptoms are exacerbated by laying in the recumbent position and associated with night-time dyspneic episodes. He is concerned about becoming short of breath when putting on clothes in the morning, but notes that he is fine when sitting and reading. Vitals reveal that his respiratory rate was 32/min upon entering the clinic but is now 16/min after resting. Physical examination reveals an extra heart sound after S2 with associated fine, basilar rales bilaterally. The most appropriate New York Heart Association classification of this patient is A. I B. II C. III D. IV

The correct answer is: C Congestive heart failure (CHF) is a clinical syndrome resulting from a mismatch between the heart's ability to pump blood and the body's circulatory demands. In systolic dysfunction, there is impaired contractility of the heart (due to cardiomyopathy, myocarditis, or after MI). In diastolic dysfunction, there is impaired ventricular filling during diastole (due to hypertension, valvular disease, or restrictive cardiomyopathy). There is a spectrum of presentations based upon severity, but most commonly, patients can present with dyspnea, orthopnea, and paroxysmal nocturnal dyspnea. There may be a nocturnal cough, cool extremities at rest, diaphoresis, or confusion as well. On cardiac exam, one could observe a displaced PMI, a pathologic S3, an S4. On respiratory exam, crackles or rales at the bases, dullness to percussion, or decreased tactile fremitus might be heard. In the case of right-sided failure, there might be peripheral pitting edema, nocturia, JVD, hepatojugular reflex, or ascites. In this patient, we observe many of the signs/symptoms. Our exam is telling as well. Further, he is tachypneic with mild exertion. The New York Heart Association (NYHA) Classification is used to quantify the degree of functional limitation imposed by CHF. It is the most commonly used system for this purpose and divides patients into categories based upon the degree of effort needed to elicit symptoms. NYHA I: symptoms only occur with vigorous activities. They are nearly asymptomatic. These levels of activity would limit normal individuals. Ex: sports. NYHA II: symptoms occur with prolonged/moderate. They are symptomatic with ordinary exertion. There is slight limitation of activities. Ex: carrying groceries, climbing stairs. NYHA III: symptoms occur with usual activities of daily living. They are symptomatic with less than ordinary exertion. There is definite limitation of activities. Ex: getting dressed, walking across a room. NYHA IV: symptoms occur at rest. They are symptomatic at mostly all times. Patients are essentially incapacitated. As this patient has dyspnea with his activities of daily living (getting dressed), he qualifies for at least NYHA III. He notes that he is asymptomatic when at rest and so he would not qualify for NYHA IV.

A 32-year-old female presents with menstrual cycles occurring only twice in the last 6 months with associated hot flashes for the last four months. A review of systems is positive for dyspareunia. Vital signs reveal a heart rate of 75/min, blood pressure of 120/82 mmHg, and respiratory rate of 14/min. Physical examination is normal except for the vaginal mucosa which is dry and thinning on pelvic exam. Which of the following would confirm the most likely diagnosis? A. cortisol B. estradiol C. FSH D. LH E. prolactin

The correct answer is: C FSH (follicle stimulating hormone) elevation in the menopausal range on 2 occasions separated by at least 1 month is diagnostic of primary ovarian failure. Primary ovarian failure occurs in women less than 40 years of age. It is characterized by oligomenorrhea/amenorrhea and signs of estrogen deficiency (hot flashes, irritability, weight gain, vaginal dryness, vaginal itching, vaginal atrophy). Patients may have a normal physical exam early in the disease process. In this patient the history findings of oligomenorrhea, hot flashes, and painful intercourse point towards an estrogen deficiency. The physical exam findings of vaginal dryness and vaginal mucosal thinning are diagnostic of atrophic vaginitis which is common in females with primary ovarian failure. Patients with primary ovarian failure may also develop osteoporosis due to the lack of estrogen. Primary ovarian failure is treated with cyclic hormone (estrogen and progesterone) replacement therapy to alleviate the symptoms and to maintain bone density.

A 17-year-old high-school senior presents to her primary care physician for a routine physical examination. Her only complaint is of throbbing headaches that have been bothering her recently. They are of moderate to severe intensity, tend to peak in several hours, and last up to two days. They tend to occur only on the right side of her head but sometimes spread to encompass the entire head. She has no visual or sensory symptoms accompanying the pain but says she cannot stand bright light when she has a headache. She sometimes awakens with a headache, but more often notices one developing during the day. Ibuprofen sometimes helps relieve the pain, especially if taken soon after the onset of the headache. She says she has been under more stress lately because she is in the process of applying to college. She has no significant past medical history and only takes combined oral contraceptives for birth control and ibuprofen for pain. Family history is non-contributory. Physical exam is mostly unremarkable, however, when you palpate the cranial vault you note decreased cranial motion about the right temporal bone. Which of the following is the most likely diagnosis? A. classic migraine B. cluster headache C. common migraine D.sinus headache E. tension headache

The correct answer is: C Greater than 90% of patients who have recurrent headache as a primary complaint when presenting to their primary care doctor have migraine headaches. Migraine headaches affect approximately 13% of adults in the United States. Migraine headaches usually begin in childhood or early adolescence, are more common in preadolescent boys, three times more common in adult women, and are very uncommon in later decades. The classical presentation of a migraine headache is a throbbing, usually unilateral headache that last between 2 and 24 hours. They usually start around the frontotemporal or ocular areas and then become diffuse as time passes. Associated features include photophobia, phonophobia, and nausea. It is not uncommon to have underlying cranial somatic dysfunction with migraine headaches, particularly involving the temporal bone on the side of dysfunction. The middle meningeal artery runs across the squamous portion of the temporal bone and this artery is thought to be involved in migraine headaches. Treating cranial somatic dysfunction with OMT may be beneficial for these patients. Answer A: Classic migraine involves a sensory aura (usually visual), consisting of flashing lights, jagged lines in peripheral vision, blind spots, or other sensory experiences such as numbness/tingling or dizziness. Common migraine by definition does not include an aura. 30% of patients with migraine fit the category of classic migraine. Answer B: Cluster headaches are another neurovascular headache characterized by intense pain near the eye with many headaches in a short period of time followed by headache-free periods. Answer D: Sinus headaches are usually caused by sinus inflammation, typically linked to viral or bacterial sinus infection. There is no evidence in the question stem to suggest this patient has any sinus problems. Answer E: Tension headaches are the most common overall headaches, but also the least distinct, and are characterized by band-like pressure around the head. The diagnosis of tension-type headache relies on negative characteristics that rule out migraine or secondary headaches.

A 65-year-old with no family contacts is required to establish a judicially-appointed guardian. A. code status B. durable power of attorney C. guarda resp altor D. living will E. res ipsa loquitor

The correct answer is: C Guarda resp altor is a guardian that is judically-appointed. Meaning a guardian over a minor or incompetent individual that has been appointed by the court system. This guardian would be able to make medical legal decisions. This guardian may be a family member, legal guardian, a sibling that is over the age of 18, or a grandparent. Answer A: Code status designates whether or not a patient would want CPR, or ACLS performed. This includes chest compressions, cardioversion/defibrillation, and medications that intervene if the patient is to lose a pulse or experience an unstable tachycardia. Answer B: Durable power of attorney legally designates a surrogate to make decisions if the patient lacks the capability. This allows the individual to make medical legal decisions if the patient is unable. A power of attorney is not judicially appointed, but rather pre-determined by the patient and then signed into action by two physicians who agree that the patient is unable to make medical decisions on their own. Answer D: A living will addresses the patient's wishes to withhold or withdraw life-sustaining medical care in the events of terminal disease or a persistent vegetative state. Answer E: Res ipsa loquitor means "the thing speaks for itself." This is a presumption that an injury would not normally occur in the absence of negligence and when its cause was under the exclusive control of the defendant. A common example would be leaving a scalpel blade inside of the patient during a surgical procedure; obviously this 'speaks for itself.'

For each numbered item (patient presentation), select the one heading (treatment) most closely associated with it. Each lettered heading may be selected once, more than once, or not at all A. citalopram B. donepezil C. memantine D. omega-3 fatty acids E. selegiline An 81-year-old Caucasian male with a history of dementia, hypertension, and hyperlipidemia presents for his annual examination. His routine medications include lisinopril, atorvastatin, and galantamine. He was diagnosed with dementia about five years ago and his MMSE score back then revealed mild dementia. However, in this visit is MMSE score is worsened to the moderate category. The patient is unable to draw a clock on the mini-mental status exam.

The correct answer is: C Memantine is a disease modifying drug used in the treatment of dementia. The NMDA receptor in the hippocampus is involved in learning and memory. Excess NMDA stimulation from ischemia can lead to excitotoxicity. Memantine is an N-methyl-D-aspartate (NMDA) receptor antagonist that is thought to be neuroprotective. In various trials it has been shown to have modest benefits in moderate to severe Alzheimer's disease. Combination with a cholinesterase inhibitor such as galantamine or donepezil can be used in advanced disease to improve symptoms.

A 43-year-old Caucasian male presents for a general maintenance examination. Past medical history is pertinent for mild intermittent asthma and a tonsillectomy at age 7. He currently uses an albuterol inhaler 1-2 times per week. His family history is positive for diabetes mellitus. The patient has a blood pressure of 122/81 mmHg, a heart rate of 77/min, and a respiratory rate of 18/min. Physical examination is unremarkable. A fasting lipid profile is obtained and reveals the following: Total cholesterol 230 mg/dL HDL 49 mg/dL LDL 165 mg/dL TRG 140 mg/dL Which of the following is the most appropriate recommendation at this time? A. HMG-CoA reductase inhibitor B. fibric acid C. lifestyle modification D. nicotinic acid E. omega-3 fatty acid

The correct answer is: C The most appropriate therapy in this 43-year-old male without known cardiovascular disease or diabetes with an LDL 165, and LDL-C >160 mg/dL and < 190 mg/dL is to initiate lifestyle modifications and recheck in 6 months. Recommendations for lifestyle modification include eating a heart-healthy diet, performing regular aerobic exercise, maintaining a desirable body weight, and abstaining from tobacco products. The HDL of 49 mg/dL is less than ideal, goal being >60 mg/dL to be a negative cardiovascular risk factor. His total cholesterol is elevated at 230 mg/dL with a goal of < 200 mg/dL. The new guidelines recommend initiating of HMG-CoA reductase inhibitors (statins) in patients with an LDL-C >190 mg/dL. In 2013, the guidelines for the treatment of hyperlipidemia in primary prevention of cardiovascular disease were updated by The American College of Cardiology and American Heart Association (ACC/AHA). The calculator uses the patient's age, gender, ethnicity, smoking status, diagnosis of diabetes, current treatment for hypertension, total cholesterol, HDL, and systolic blood pressure to calculate the 10-year cardiovascular risk. The ACC/AHA made the following recommendations for adults ages 40-75 without known cardiovascular disease (CVD) and LDL-C between 70 mg/dL and 189 mg/dL based on risk calculation using the ACC/AHA calculator: • In those without diabetes: -Treat those with an estimated 10-year CVD risk ≥ 7.5% with moderate- to high-intensity statin therapy. -It is reasonable to offer treatment with moderate intensity statin therapy to those with an estimated 10-year CVD risk between 5-7.5%. • In those with diabetes: -Treat with at least moderate statin therapy. -High-intensity statin therapy is reasonable in those with an estimated 10-year CVD risk ≥ 7.5 %. Answer A: According to the ATP III guidelines it is not recommended for him to start on a HMg-CoA reductase inhibitor. According to ATP III guidelines, the indication for starting pharmaceutical therapy with HMG-CoA reductase inhibitor in a patient with 0-1 risk factors is an LDL >190 mg/dL. It recommends initiating lifestyle modifications at an LDL >160 mg/dL. This patient has 0 risk factors and thus has likely has a 10-year risk of cardiovascular disease <10% and there is no recommendation for initiating pharmaceutical therapy initially. Answer B: Fibric acid therapy (Gemfibrozil, Fenofibrate, Clofibrate) is recommended as adjunctive therapy for patients with severe hypertriglyceridemia. The first line therapy for hypertriglyceridemia is a HMG-CoA reductase inhibitor. According to the new 2013 guidelines, fibrates can be added to HMG-CoA reductatse therapy to lower triglyceride levels in high risk patients. This patient has normal triglyceride level and therefore does not need treatment for hypertriglyceridemia. Answer D: Niacin therapy has been shown to increase HDL levels. However, it has not shown to have a mortality or morbidity benefit in the primary prevention of cardiovascular disease and therefore it is not recommended for treatment of low HDL. The recommendation instead is to increase consumption of omega-3 fatty acids, soluble fiber, maintenance of healthy weight, and increase in physical aerobic activity. Answer E: Omega-3 fatty acid supplementation is indicated for adjunctive treatment of hyperlipidemia not at goal with high intensity HMG-CoA reductase inhibitor therapy in the secondary prevention of cardiovascular events. It is not recommended for the primary prevention of cardiovascular disease.

A 55-year-old obese female with past medical history of rheumatoid arthritis and hypertension presents for routine physical examination. Medications include methotrexate, prednisone, and lisinopril. Vitals include a temperature of 37.0ºC (98.6ºF), heart rate of 87/min, respiratory rate of 13/min, and blood pressure of 160/85 mm Hg. Physical examination includes thin skin with multiple bruises, a round plethoric face, and violaceous abdominal striae. Cardiovascular examination is unremarkable. There is trace pitting edema to the bilateral lower extremities. You note a tender nodule on the left one inch lateral and two inches superior to the umbilicus. Which of the following laboratory abnormalities is most likely? A. elevated TSH B. elevated vanillylmandelic acid C. hypoglycemia D. hypokalemia E.leukopenia

The correct answer is: D Iatrogenic glucocorticoid overuse including prednisone is the most common cause of Cushing's disease. Common clinical manifestations include proximal muscle weakness, easy bruising, weight gain, and hirsutism. Common physical examination findings include hypertension, moon facies, posterior cervical fat pad, and truncal obesity as present in this patient. Common laboratory abnormalities include hyperglycemia, hypokalemia and leukocytosis. Hypokalemia is the most common electrolyte abnormality noted in patients with Cushing's syndrome because of increased mineralocorticoid activity in patients with high cortisol levels. The tender nodule 2" superior and 1" lateral to the umbilicus is classic for a Chapman point for the adrenal glands. Answer A: Elevated thyroid stimulating hormone (TSH) levels are seen in patients with primary hypothyroidism. Patients with this condition may likewise present with weight gain; however, the additional findings of hypertension, thin skin, and dark purple striae are more consistent with Cushing's syndrome rather than hypothyroidism. Answer B: Elevated vanillylmandelic acid (VMA) is found in patients with pheochromocytoma. Although patients with pheochromocytoma do present with hypertension and tachycardia along with palpitations and diaphoresis, truncal obesity and purple striae are not consistent with this diagnosis. Answer C: Excess cortisol results in stimulation of gluconeogenesis as well as peripheral insulin resistance resulting in hyperglycemia instead of hypoglycemia. Answer E: Excess cortisol secondary to cushings syndrome results in leukocytosis instead of leukopenia. Myelosuppression is the major dose-limiting side effect of high-dose methotrexate, but is infrequent in patients receiving low-dose therapy. With low-dose weekly therapy, as used in rheumatoid arthritis, hematologic toxicity in association with macrocytic red blood cells may be seen, but a more serious abnormality is the development of pancytopenia.

A 30-year-old man with no previous medical history presents to his primary care physician because of dizziness of several weeks duration. He reports that the dizziness is worst in the morning when he gets out of bed, but can occur at other times as well. He describes the room spinning around him for about 30 seconds. The symptoms resolve if he sits still. During this time, he feels slightly nauseated but he reports no episodes of vomiting. He denies headache, ear pain or fullness, difficulty hearing, tinnitus, visual difficulties, weakness, and changes in sensation. When the patient is tilted back on the examination table with his head rotated, he complains of dizziness, and left-beating nystagmus is observed. The remainder of the examination is unremarkable. Which of the following is the most appropriate treatment for this patient? A.brain MRI B. Dix-Hallpike maneuver C. Epley maneuver D. meclizine E. watchful waiting

The correct answer is: C This patient's clinical presentation and exam findings are most consistent with benign paroxysmal positional vertigo (BPPV), which is the most common cause of vertigo. BPPV is characterized by fleeting episodes of vertigo that most commonly occur when the person changes position, especially head position. Patients typically note this sensation immediately upon waking. The maneuver described in the stem is the Dix-Hallpike maneuver, which is diagnostic for the condition. Patients will complain of their symptoms and will have nystagmus during the maneuver. BPPV is caused by abnormal positioning of the otoliths within the semicircular canals. The Epley maneuver is a method of canalith repositioning. It has an extremely favorable risk/benefit ratio and is the first line of treatment for BPPV. The patient is moved through a series of positions that attempt to move the otolith out of the semicircular canal. Treatment for BPPV also includes vestibular rehabilitation, which involves balance exercises and physical therapy.

17-year-old female presents to the physician with a 1-year history of fatigue, cramping abdominal pain, and watery diarrhea. She has also experienced an 8.2-kg (18-lb) weight loss over the past 6 months. Physical examination reveals right lower quadrant tenderness and fullness and several aphthous ulcers in her mouth. Complete blood count reveals a hemoglobin of 10.1 g/dL and mean corpuscular volume (MCV) of 72 fL. The most appropriate diagnostic modality to confirm the diagnosis is A. abdominal CT scan B. barium enema C. capsule endoscopy D. colonoscopy E.ultrasound of the right lower quadrant

The correct answer is: D A history of recurrent abdominal pain, diarrhea, and weight loss in a child or young adult is characteristic of inflammatory bowel disease. Crohn's disease typically present in adolescent patients with fever, weight loss, and abdominal pain with frequent, sometimes bloody diarrhea. This is the result of chronic transmural inflammation of the bowel, which usually involves the terminal ileum (hence RLQ pain), but may involve any intestinal tissue between the mouth and the anus. Crohn's disease is a chronic, indolent disease characterized by unpredictable flares and remissions of symptoms. 15-20% of patients experience extraintestinal symptoms including uveitis, arthritis, ankylosing spondylitis, erythema nodosum, pyoderma gangrenosum, aphthous oral ulcers, and cholelithiasis). Definitive diagnosis is made via endoscopic visualization and tissue biopsy of involved intestinal mucosa. Colonoscopy with intubation of the terminal ileum is used to evaluate the extent of disease, to demonstrate strictures and fistulae, and to obtain biopsy samples. Characteristic histologic findings include skip lesions, noncaseating granulomas, transmural thickening and inflammation with narrowing of the lumen, and mesenteric "fat creeping" onto the antimesenteric border of the small bowel. Treatment includes anti-inflammatory agents (sulfasalazine, prednisone) and/or immunosuppressants (azathioprine, 6-mercaptopurine, methotrexate). However, the effectiveness of medical treatment decreases with advancing disease and complications (e.g., fistulas, strictures, bowel obstruction) eventually develop, requiring surgery (segmental bowel resection). There is no cure and recurrence usually occurs after surgery.

A 65-year-old presents a document that addresses wishes to withhold or withdraw life-sustaining medical care in the event of terminal disease or a persistent vegetative state. A. code status B. durable power of attorney C. guarda resp altor D. living will E. res ipsa loquitor

The correct answer is: D A living will is a legal document that addresses the patient's wishes to withhold or withdraw life-sustaining medical care in the events of terminal disease or a persistent vegetative state. This document allows medical care personnel to treat the patient how the patient would want in the event of a terminal disease. Answer A: Code status designates whether or not a patient would want CPR and/or intubation performed. This includes chest compressions, cardioversion, and medications that intervene if the patient is to lose a pulse or experience an unstable tachycardia. Code status is generally not a legal document per se because it can change from day-to-day or hospitalization-to-hospitalization. Also the status supplements existing documents such as advanced directives or living wills rather than replaces them. A code status has no implications on the medical care of the terminally ill or those in a persistently vegetative state, which the question refers to. Answer B: Durable power of attorney legally designates a surrogate to make decisions if the patient lacks the capability. This allows the individual to make medical legal decisions if the patient is unable. Answer C: Guarda resp altor is a guardian that is judically-appointed. Meaning a guardian over a minor that has been appointed by the court system. This guardian would be able to make medical legal decisions. This guardian may be the minor's parent, legal guardian, a sibling that is over the age of 18, or a grandparent. Answer E: Res ipsa loquitor means "the thing speaks for itself." This is a presumption that an injury would not normally occur in the absence of negligence and when its cause was under the exclusive control of the defendant. A common example would be leaving a scalpel blade inside of the patient during a surgical procedure; obviously this 'speaks for itself.'

A 30-year-old female presents for a follow-up appointment for her Celiac disease which was diagnosed two weeks ago. History reveals she still has recurrent diarrhea but now denies nausea, vomiting, or abdominal cramping. Which of the following is the most appropriate next step in management for this patient? A. obtain a food allergy panel B. repeat anti-gliadin antibody titer C.repeat IgA anti-tissue transglutamine antibody titer D. replace gluten food groups with buckwheat, corn, potatoes, and rice E. start prednisone 20 mg daily for 14 days

The correct answer is: D Celiac disease is defined as a gluten sensitive enteropathy and strict gluten avoidance is key to therapy. Principle sources of dietary gluten include wheat, rye, barley, most beers, malt vinegars, and oats, however, small amounts of gluten is present ubiquitously in many food products. Soybean or tapioca flours, rice, corn, buckwheat, and potatoes are safe. A strict gluten free diet will result in resolution of symptoms in greater than 90% of the patients. Approximately 70% of people will have a response within two weeks. Those effected typically present with diarrhea and malabsorptive symptoms like iron deficiency anemia, vitamin/mineral deficiency, osteopenia, neurologic symptoms. Non-compliance with diet restrictions is the most common cause of recurrent symptoms of celiac disease making dietary modification the correct answer choice. Monitoring the response to a gluten-free diet with serology testing or a gluten challenge is not recommended until at least 3 months from attempting lifestyle modifications.

A 29-year old male with past medical history of asthma and treated gonorrheal infection presents with a copious amount of clear discharge from the eyes bilaterally. He is concerned because he feels as though something is in his eyes and because he wakes up with his eyelids sticking closed together. He remarks that his eyes have been itchy and red with constant tearing. He had a low-grade fever, cough, and runny nose which resolved about three weeks ago. Physical exam reveals injected conjunctivae. Question 1 of 2 in this set The most likely diagnosis is A. adenoviral conjunctivitis B.chlamydial conjunctivitis C. gonococcal conjunctivitis D. seasonal allergic rhinoconjunctivitis E. streptococcal conjunctivitis

The correct answer is: D Conjunctivitis is the most common cause of a red, irritated eye. It is defined as an inflammation of the palpebral conjunctiva (the transparent membrane that lines the inside of the eyelids) and the globe (bulbar conjunctiva). Most cases are viral, though bacterial, allergic, or chemical irritants are other etiologies. Infectious conjunctivitis is usually spread through direct contact or transmission via a fomite. Pain should typically only be minimal and visual acuity may be reduced, but should only be slightly so. As many forms of conjunctivitis exist and the treatment for each varies greatly, it is important to tease out the etiology based on close attention to the history. Typically, the type of discharge and the involvement of one or both eyes can tell us a great deal. Viral conjunctivitis causes a watery discharge, mild foreign-body sensation, and photophobia. Bacterial infection differs in that it produces a mucopurulent discharge (rather than clear). Allergic conjunctivitis can present similar to adenoviral conjunctivitis in that a bilateral watery discharge is observed, but it differs in that pruritus and a foreign body sensation are unique to allergic causes. His morning crusting due to exudate would be suspicious for bacterial causes, but then this exudate should be mucopurulent. The patient also has an atopic condition, which also points to allergic causes. The fact that he had a viral prodrome would make us consider adenovirus, but this is typically observed simultaneously to the conjunctivitis or two weeks before it, at the most. His previous history of sexually-transmitted disease is tempting to consider, but it was treated and there is no other indication in the history of bacterial involvement. Noninfectious causes of conjunctivitis include allergic and chemical irritants. Allergic conjunctivitis features bilateral itching, tearing, redness, stringy discharge, and even photophobia. A foreign body sensation is also classic. On examination, cobblestone papillae can be seen and this is also seen when the etiology is contact lenses (or any other chronic foreign body). Treatment is also usually symptomatic, with oral antihistamines or topical antihistamine or anti-inflammatory drops. Cold compresses (rather than warm in viral conjunctivitis) can be helpful and removal of the allergen is attempted. It is very common among patients with atopic disease and usually presents seasonally. Conjunctivitis secondary to irritants can be caused by contact lenses, chemicals, foreign bodies, and even dryness (keratoconjunctivitis sicca). Answer A: Epidemic keratoconjunctivitis is very contagious and spread by person-to-person contact or fomites. The most common cause is adenovirus. There is usually an associated pharyngitis, fever, malaise, and preauricular lymphadenopathy is observed on physical exam. However these symptoms are typically concurrent or conjunctivitis can follow up to two weeks later (more time has passed in this patient). On exam, the palpebral conjunctiva is erythematous and copious watery discharge with some scant exudates can be observed. Some may start as unilateral but then spread to the other eye in a few days. Topical antibiotics can be used to prevent secondary bacterial infection and steroids are used for membranous conjunctivitis but otherwise, hot compresses are used for symptomatic treatment. Answer B: There are actually two forms of chlamydial conjunctivitis. Trachoma is caused by Chlamydia trachomatis serotypes A, B, and C and is the most common cause of blindness worldwide, as it causes chronic scarring under the eyelid. Though it is not seen commonly in the United States, it is very common in developing nations. Inclusion conjunctivitis is caused by C. trachomatis serotypes D to K and is transmitted by genital-hand-eye contact in patients with the STD. Symptoms are similar to those of bacterial conjunctivitis though this form differs from trachoma in that it does not cause corneal scarring and subsequent blindness. Adults and teens are typically treated with oral tetracycline, doxycycline, or erythromycin for about two weeks, and sexual partners are treated as well. Answer C: Hyperacute bacterial conjunctivitis is caused by Neisseria gonorrhoeae and should be considered in a patient who presents with rapid onset of copious, purulent exudate. Symptoms will rapidly progress to severe redness, swelling, and pain. Patients are typically sexually active young adults and the concern here is the development of corneal scarring and blindness when this process is left untreated. Therefore, immediate ophthalmologic referral is required in these cases Gonococcal conjunctivitis is treated with a one-time dose of ceftriaxone 1g IM, as well as topical therapy as per an ophthalmologist. Though this patient is a young adult who we know is sexually-active, his infection was previously treated. The history also does not suggest any bacterial involvement. Answer E: The most common bacterial causes of conjunctivitis include Staphylococci, Streptococci, Haemophilus, Moraxella, and Pseudomonas. There is a rapid onset of irritation, hyperemia, and tearing with spread to the other eye in less than 2 days. Exudate is usually mucopurulent, with crusting apparent. Patients are typically treated empirically with broad-spectrum antibiotics (erythromycin, ciprofloxacin, sulfacetamide) and as in adenoviral conjunctivitis, strict observation of personal hygiene must be observed to prevent spread. Conjunctival cultures can be taken if a patient is resistant to broad-spectrum coverage or the disease process is particularly severe.

A 55-year-old man presents for a health maintenance examination. History reveals that he was told that he had diabetes several years ago, although he never follow-up with a physician after that time. Physical examination reveals his vision is 20/40 bilaterally, uncorrected, with intact extra-ocular muscles and pupils that are equal, round, and reactive to light and accommodation. You note a tender nodule on the anterolateral aspect of the right humeral head. Which of the following finding on the funduscopic exam would be most specific to diabetic retinopathy? A. arterioveneous nicking B. cotton wool spots C.microaneurysm D. neovascularization E. retinal hemorrhage

The correct answer is: D Diabetic retinopathy can be separated into two categories: non-proliferative and proliferative retinopathy. Non-proliferative retinopathy consists of nerve-fiber layer infarcts (cotton-wool spots), intra-retinal hemorrhages, and development of macular edema. Proliferative retinopathy involves neovascularization of the retinal vessels. Neovascularization is stimulated by vascular endothelial growth factor, which is released after the initial cell death due to ischemic injury. Proliferative retinopathy can result in vitreous hemorrhage and retinal detachment. The sequence of events leading to proliferative retinopathy is specific to diabetic retinopathy. A chapman point on the anterolateral humeral head may be present with any pathology involving the retina or inner chamber of the eye. Answer A: Arteriovenous nicking is more commonly seen in hypertensive retinopathy. Answers B & C & E: Cotton wool spots, retinal hemorrhage, and microaneurysm can occur in both diabetic and hypertensive retinopathy; therefore, it is not specific to diabetic retinopathy.

For each numbered item (patient presentation), select the one heading (treatment) most closely associated with it. Each lettered heading may be selected once, more than once, or not at all A. citalopram B. donepezil C. memantine D. omega-3 fatty acids E. selegiline A 34-year-old healthy female presents to her primary care physician for her annual physical examination. She does not have any physical complaints today and appears to be healthy on physical examination. She does not have any known medical conditions. She does not endorse any psychiatric or mood symptoms. She works as an elementary school teacher and lives alone. She enjoys traveling, reading, and hiking. Family history is positive for Alzheimer's disease, Parkinson's disease and coronary artery disease. Routine labs are performed and are within normal limits, including CBC, CMP, TSH, and urinalysis.

The correct answer is: D Omega-3 fatty acids are polyunsaturated fatty acids that are essential for normal human metabolism. There have been proposed health benefits including anti-inflammatory activities, lipid lowering effects and reduction in systolic blood pressure. Omega-3 fatty acids are found in fish, walnuts and eggs. Support from observational studies has shown that omega 3 supplementation can lower the risk of developing dementia. However, there lacks evidence based medicine showing a benefit of omega-3 supplementation in treatment of Alzheimer's disease. Therefore, omega-3 fatty acids supplementation would not be appropriate in a patient with preexisting dementia

A 30-year-old marathon runner presents with left lower extremity pain associated with ambulation of four months' duration. History reveals that he has signs of claudication after walking two blocks. Physical examination reveals palpable pedal pulses. A duplex ultrasound is obtained and reveals elevated velocities in the popliteal artery with no other abnormalities. Which of the following popliteal artery dysfunctions is causing his symptomology? A. aneurysm B. atherosclerosis C. embolus D. entrapment E. thrombus

The correct answer is: D Popliteal artery entrapment is a rare cause of intermittent claudication or chronic lower extremity pain in young adults. These patients are typically male, have claudication associated with movement, and can have palpable pedal pulses. Symptoms may progress to paresthesias as they develop fibrosis of the popliteal artery. Popliteal entrapment develops in utero during the migration and development of either the popliteal artery or medial head of the gastrocnemius muscle. Answer A: Popliteal artery aneurysms can cause symptoms of claudication due to showering emboli to the tibial vessel, however, the patient has palpable pedal pulses and a normal ultrasound which rules out popliteal artery aneurysm. Answers B & C & E: Atherosclerosis, a thrombus, or an embolus are all unlikely in this young adult patient. Atherosclerosis is unlikely simply due to the patient's age whereas a thrombus or embolus is incorrect due to the acuity associated with them.

A 25-year-old male presents bilateral eye irritation for three days. He states he has been studying for a medical school exam all week and fell asleep multiple times without removing his contacts. He denies vision changes or pain, but does admit to a gritty sensation when he blinks and thick green discharge. Physical examination reveals pupils equal, round, and reactive to light with intact extraocular motions. You note paravertebral muscle spasms from T1-4 with tenderness to palpation noted in this region. Slit-lamp exam shows a visible fluorescein uptake on the cornea. Which of the following is the most likely complication of this condition? A. blepharitis B. cataracts C. open-angle glaucoma D. Pseudomonas infection E. uveitis

The correct answer is: D Prolonged use of contact lens prevents the eye from adequate hydration, which can lead to corneal abrasions and increase the risk for subsequent infection. Eye infections are often due to staphylococcus species, but the most common bacteria seen in contact lens wearers is pseudomonas. Corneal abrasions are diagnosed by fluorescein exam showing green branching when viewed with cobalt blue filter on slit-lamp examination. The eyelid should also be examined to evaluate for foreign bodies. Treatment of a simple corneal abrasion is topical anti-inflammatories such as diclofenac or ketorolac. Steroids are contraindicated. Eye patches have not been shown to be effective. Expected time course is 48-72 hours. If infected is noted, then anti-pseudomonal antibiotics such as ciprofloxacin, ofloxacin, or gentamicin should be used. Contact use should be discontinued until abrasion heals and antibiotic course is complete. As with any pathology in the head and neck region, viscerosomatic reflexes may result in TART changes present from T1-4. Answer A: Blepharitis is inflammation of the eyelid, and often associated with staphylococcus species. It is an external eye infection, so patients may complain of discharge and sometimes burning, but no gritty sensation. Eyelid exam may show lid swelling and crusting on the lashes. Slit-lamp exam would be normal. Treatment involves warm compresses, lid scrubbing, and hand hygiene. Severe cases can be treated with antibiotics. Answer B: Cataracts are opacifications of the natural lens, and is commonly seen in elderly patients. It presents as a gradual onset of loss of visual acuity over many years. While it can be associated with trauma, it is not an acute condition. Other risk factors include cigarette smoking, steroid use, prolonged radiation exposure, Down syndrome, and Wilson's disease. Exam will show white cloudy appearance of the lens, otherwise a normal slit lamp exam. Treatment involves surgical removal of the affected lens and replacement with an artificial intraocular lens. Answer C: Glaucoma is due to increased intraocular pressure, and comes in two types: open angle and closed angle. It is important to identify because it can lead to blindness. Open angle glaucoma is due to impaired outflow of the aqueous humor. It is painless and causes gradual visual field loss with central sparing. Closed angle glaucoma is obstruction of the outflow, can cause rapid onset of redness and pain, and should be promptly treated to prevent blindness. Bilateral simultaneous closed angle glaucoma is rare. The affected pupil will be dilated and nonreactive. Both types of glaucoma are diagnosed by tonometry to check eye pressure, and visual field exam. Answer E: Uveitis is inflammation of the uvea, which consists of the vascular structures of the eye including the iris and ciliary body. It is associated with connective tissue diseases, and presents with blurry vision, pain, photophobia, redness around the cornea, and a constricted pupil in the affected eye compared to the contralateral eye. Uveitis should be referred urgently to an ophthalmologist, and is often treated with topical steroids after a corneal abrasion has been ruled out.

A 41-year-old male complains of colicky abdominal pain, tingling in his fingers and toes, and intermittent confusion. Physical exam reveals a chronically ill appearing gentleman with decreased visual acuity, slight scleral icterus and jaundice. Lab values reveal anemia. Further history reveals that he works at a battery factory and suspects that he has been exposed to lead. You should report this to the A. CDC B. FDA C. NIH D. OSHA E.USDA

The correct answer is: D This patient has suffered an untoward detriment to his health as a result of his occupational exposure to lead. Battery manufacturing (along with chemical production, construction and demolition workers, firing-range instructors, foundry workers, gas-station attendants, jewelers, pigment manufacturing, pipefitters, plastics industry, pottery workers, printers, radiator repair, rubber industry, stained-glass makers, and welders) are at risk for long-term low-level exposure (or short-term high-level exposure) to lead. Some of the associated findings are colicky abdominal pain, encephalopathy, fatigue, decreased visual acuity (due to lead-induced optic neuropathy), hemoglobinuria, hemolytic anemia with resultant jaundice and splenomegaly. The Occupational Safety and Health Administration, a division of the United States Department of Labor, is responsible for monitoring and addressing occupational exposures (both incidental and negligent) of employees to hazardous materials or conditions. OSHA is responsible for the regulation of lead exposure in all industries in the United States. Appropriate medical therapy should be provided along with notification of the appropriate agency. Answer A: The CDC does keep account of lead poisoning statistics and provides information for those individuals at risk for lead poisoning. However, the CDC would be the best agency to contact if this patient encountered their lead exposure in the general public, such as older housing with lead water pipes or lead-based paints. Answer B: The FDA is responsible for overseeing medical products and tobacco, foods, global regulatory operations and policy, and operations. It is a division of the Department of Health and Human Services. It is not directly responsible for the occurrence reporting of occupational exposures. Answer C: The NIH was founded with the goal of seeking and understanding the impact of pathogens and our environment on our health as a nation and to encourage research in the areas of treating diseases we face. A great deal of work has been done at the NIH regarding lead poisoning, but it is not the appropriate agency to contact in the event of an individual's occupational lead exposure. Answer E: The USDA provides leadership on food, agriculture, natural resources, rural development, nutrition, and other related issues. The focus is on the progress of the nation's agricultural industry and the safety thereof. The USDA does not deal directly with occupational lead exposure. However, the USDA would be the agency to contact if there were an outbreak of meat or produce that was found to have high levels of lead.

A 16-year-old female presents with a one-week history of fatigue, dull abdominal pain and sore throat. She has no major medical history and reports she is not sexually active. Her temperature is 38.9°C (102°F), blood pressure is 118/80 mmHg, pulse is 65/min, and respiratory rate is 12/min. Physical examination reveals diffuse pharyngeal inflammation, tonsillar exudates, splenomegaly, and tender cervical lymphadenopathy. Which of the following is the most definitive diagnostic modality? A. abdominal CT scan B. complete blood count with differential C. fine needle aspiration of a lymph node D. latex agglutination assay E. rapid Strep antigen detection

The correct answer is: D This patient is presenting with signs and symptoms classic for infectious mononucleosis caused by Epstein-Barr virus (EBV). EBV is usually transmitted by saliva and infects the epithelial cells of the oropharynx and nasopharynx. The virus can remain in the saliva for several months after it is contracted, and approximately 25-50% of patients exposed to EBV carry the virus without experiencing clinical disease. Clinical disease, termed infectious mononucleosis, occurs after viral dissemination to the peripheral blood lymphocytes and plasma. It is commonly seen in young adults 15 to 25 years of age, but can occur at any age. Early infection is usually asymptomatic, but may be associated with a prodrome of headache, malaise and fatigue. Active disease commonly presents with severe sore throat, fever, malaise, myalgia, and anorexia. Physical findings include diffuse pharyngeal inflammation, edema, enlarged tonsils with exudates, and tender lymphadenopathy. Lymphadenopathy is most prominent in the anterior and posterior cervical chains. Other findings include splenomegaly (50%), hepatomegaly (10-20%), and maculopapular rash (5-15%). The "Monospot" test is a latex agglutination assay that contains horse red blood cells, which agglutinate when exposed to heterophile antibodies (type of antibodies produced in response to EBV). This test has a sensitivity of 85% and specificity of 100% for diagnosing EBV infection. False-negative "mono spot" tests may occur in children less than 2-4 years of age and in early stages of EBV infection (heterophile antibodies usually present and peak 2-6 weeks after initial infection). False-positive results may be seen in patients with toxoplasmosis, CMV, rubella, lymphoma, SLE, rheumatoid arthritis, HIV, HSV and certain malignancies (leukemia, lymphomas). If false results are expected then testing for more specific EBV antibodies should be conducted. Treatment is usually supportive with rest, fluids, and analgesics. Glucocorticosteroids can be used in patients with significant tonsil enlargement (to prevent respiratory compromise), thrombocytopenia, and hemolytic anemia.

A 16-year-old female presents with a one-week history of fatigue, dull abdominal pain and sore throat. She has no major medical history and reports she is not sexually active. Her temperature is 38.9°C (102°F), blood pressure is 118/80 mmHg, pulse is 65/min, and respiratory rate is 12/min. Physical examination reveals diffuse pharyngeal inflammation, tonsillar exudates, splenomegaly, and tender cervical lymphadenopathy. Question 1 of 2 in this set Which of the following is the most definitive diagnostic modality? A. abdominal CT scan B. complete blood count with differential C. fine needle aspiration of a lymph node D. latex agglutination assay E. rapid Strep antigen detection

The correct answer is: D This patient is presenting with signs and symptoms classic for infectious mononucleosis caused by Epstein-Barr virus (EBV). EBV is usually transmitted by saliva and infects the epithelial cells of the oropharynx and nasopharynx. The virus can remain in the saliva for several months after it is contracted, and approximately 25-50% of patients exposed to EBV carry the virus without experiencing clinical disease. Clinical disease, termed infectious mononucleosis, occurs after viral dissemination to the peripheral blood lymphocytes and plasma. It is commonly seen in young adults 15 to 25 years of age, but can occur at any age. Early infection is usually asymptomatic, but may be associated with a prodrome of headache, malaise and fatigue. Active disease commonly presents with severe sore throat, fever, malaise, myalgia, and anorexia. Physical findings include diffuse pharyngeal inflammation, edema, enlarged tonsils with exudates, and tender lymphadenopathy. Lymphadenopathy is most prominent in the anterior and posterior cervical chains. Other findings include splenomegaly (50%), hepatomegaly (10-20%), and maculopapular rash (5-15%). The "Monospot" test is a latex agglutination assay that contains horse red blood cells, which agglutinate when exposed to heterophile antibodies (type of antibodies produced in response to EBV). This test has a sensitivity of 85% and specificity of 100% for diagnosing EBV infection. False-negative "mono spot" tests may occur in children less than 2-4 years of age and in early stages of EBV infection (heterophile antibodies usually present and peak 2-6 weeks after initial infection). False-positive results may be seen in patients with toxoplasmosis, CMV, rubella, lymphoma, SLE, rheumatoid arthritis, HIV, HSV and certain malignancies (leukemia, lymphomas). If false results are expected then testing for more specific EBV antibodies should be conducted. Treatment is usually supportive with rest, fluids, and analgesics. Glucocorticosteroids can be used in patients with significant tonsil enlargement (to prevent respiratory compromise), thrombocytopenia, and hemolytic anemia. Answer A: Patients with infectious mononucleosis commonly experience dull abdominal pain due to splenic and possibly hepatic enlargement, leading to abdominal congestion. If her latex agglutination assay was negative and her abdominal pain increased or persisted, it would be reasonable to perform abdominal imaging. However, it is not necessary to perform an abdominal CT for infectious mononucleosis. Answer B: Complete blood count with differential should be obtained in all patients presenting with symptoms of mononucleosis. EBV infection is characterized by a lymphocytosis with atypical lymphocytes (reactive cytotoxic T cells that are attacking virally-infected host cells). Presence of a normal or low WBC count can help to rule out EBV infection. However, an elevated white count and the presence of atypical lymphocytes are nonspecific findings. Atypical lymphocytes can be seen in several other infections (e.g., rubella, measles, mumps, hepatitis, CMV, HIV) and, therefore, cannot be used to diagnose infectious mononucleosis. Other possible findings on CBC with EBV infection include thrombocytopenia and rarely, hemolytic anemia (autoimmune with cold agglutinins). Other tests to consider in patients with suspected EBV infection include liver function studies (commonly have elevated aminotransferases and bilirubin) and erythrocyte sedimentation rate (elevated in EBV vs. normal in streptococcal pharyngitis). Answer C: Fine-needle aspiration is a minimally invasive technique used to diagnose diseased tissue. It usually is reserved for patients with persistent lymphadenopathy, lymph nodes larger than 3 cm in diameter, or nontender lymphadenopathy where there is concern for possible malignancy. It would be overly aggressive in this patient who is presenting with a more acute, infectious process. Answer E: A rapid strep antigen detection test can be used in the out-patient setting to detect a group A streptococcal infection in 10 to 15 minutes. It is commonly used with a throat culture to make the diagnosis of streptococcal pharyngitis. This patient's presentation is more consistent with the diagnosis of infectious mononucleosis due to the presence of abdominal pain and splenomegaly, which are not present with strep pharyngitis. However, patients with EBV infection are at risk of developing a secondary pharyngeal infection with beta-hemolytic streptococci. Therefore, if a patient with infectious mononucleosis experiences persistent or worsening symptoms consider evaluating for streptococcal infection. If secondary bacterial pharyngitis is present, antibiotic treatment with penicillin or erythromycin is required. Ampicillin and amoxicillin should be avoided in patients with EBV infection because they can cause a diffuse, erythematous rash.

A 70-year-old female who is 3 days status-post urologic surgery develops a fever overnight in the hospital. She has an indwelling Foley catheter with good urine output. Her past medical history includes hypertension and urinary incontinence. She has no significant family history. Vitals reveal her temperature to be 39.0°C (102.2°F), pulse at 82/min, respirations at 15/min, and blood pressure of 134/85 mmHg. Physical examination is remarkable for mild suprapubic tenderness. You note a tender nodule on the lateral border of the umbilicus. Urinalysis reveals 60 WBC/hpf, trace protein, positive leukocyte esterase, positive nitrites, and no casts. A reflex urine culture grows motile, gram-negative rods with a dark green appearance and fruity odor. The most appropriate pharmacologic therapy includes A. amoxicillin B. cefotaxime C. ceftriaxone D. ciprofloxacin E. trimethoprim-sulfamethoxazole

The correct answer is: D This woman has a urinary tract infection (UTI) due to Pseudomonas aeruginosa. Pseudomonas aeruginosa is a motile, gram negative rod that is easily identified by its green or blue pigment production and fruity odor. Urinary tract infections caused by P. aeruginosa are most commonly seen in patients with a foreign body in the urinary tract or who have obstruction of the genitourinary system. The presence of an indwelling catheter is a risk factor for UTI development as these catheters provide a seeding ground for bacteria to grow and ascend the urinary tract. As dysuria and urinary frequency may not be experienced, signs of UTI in this patient population include suprapubic tenderness, fever, and leukocytosis. P. aeruginosa cystitis is considered a complicated infection and requires longer and more aggressive antibiotic treatment than uncomplicated cystitis. Management includes removal of the foreign instrument or obstruction and a 7-10 day course of antibiotics with pseudomonal coverage. The drug class of choice is a fluoroquinolone, most commonly ciprofloxacin or levofloxacin. Remember to use your structural exam findings to help you make a diagnosis. In this case you are provided with the anterior chapman point for the bladder which is classically described and being present in the tissue surrounding the umbilicus. Answer A: The Infectious Disease Society of America (IDSA) advises against using amoxicillin or ampicillin for empiric treatment of uncomplicated or complicated cystitis due to their relatively poor efficacy and high rates of resistance. Answer B: Cefotaxime is a third generation cephalosporins with poor activity against pseudomonal infections. Ceftazidime (third generation) and cefepime (fourth generation) are the only cephalosporins that have shown to be effective against pseudomonas infections. However, their use is usually reserved for more critical infections, such as bacteremia, endocarditis, osteomyelitis, malignant otitis externa and central nervous system infections. Answer C: Ceftriaxone is a third generation cephalosporins with poor activity against pseudomonal infections. Ceftazidime (third generation) and cefepime (fourth generation) are the only cephalosporins that have shown to be effective against pseudomonas infections. However, their use is usually reserved for more critical infections, such as bacteremia, endocarditis, osteomyelitis, malignant otitis externa and central nervous system infections. Answer E: Trimethoprim-sulfamethoxazole is usually the treatment of choice for uncomplicated UTIs. However, it is not appropriate for Pseudomonas aeruginosa infections.

A 54-year-old Caucasian female with a history of asthma presents for a general wellness exam. Her asthma has been well-controlled other than an occasional cough at night. She notes an unintentional 2.26 kg (5 lb) weight loss over the last three months. Her medications include albuterol as needed and a daily multivitamin. Physical examination reveals a .5 x .8 cm firm, non-tender, non-mobile mass in the left lobe of the thyroid. No cervical lymphadenopathy is noted. Examination of other systems is otherwise unremarkable. A serum thyroid stimulating hormone is found to be 3.8 mcU/mL. Question 1 of 2 in this set Which of the following is the most appropriate next step in management? A. CT scan of the neck B. fine needle aspiration C.free T4 D. thyroid scintigraphy E. ultrasonography

The correct answer is: E Thyroid nodules are fairly common and have a lifetime prevalence of approximately 5-10%. Typically nodules greater than 1 cm are palpable on physical examination but location and anatomy of nodules can make it difficult to detect. Factors increasing the risk of malignancy include patient age (younger than 20 or older than 70), male sex, dysphagia, history of neck irradiation, firm, hard, or immobile nodule, and presence of cervical lymphadenopathy. The first step in evaluation of thyroid nodules is measurement of serum TSH to evaluate for hyperthyroidism or hypothyroidism. If TSH is normal, the next step in evaluation is ultrasound to evaluate for size and anatomic spread. For nodules greater than 1 cm, FNA is the next best step in diagnosis to rule out malignancy. Answer A: Thyroid nodules are occasionally found incidentally on CT scans; however, ultrasound is the preferred imaging technique for patients with isolated thyroid nodules found on examination. CT will also administer unnecessary radiation compared to ultrasound. Answer B: Thyroid ultrasound should be performed in all patients with a suspected thyroid nodule or nodular goiter on physical examination or with nodules incidentally noted on other imaging studies. It is a non-invasive test and may alter the management in this patient. Fine needle aspiration would be the next step after obtaining a radionuclide thyroid scan if the nodule was found to be nonfunctional or if ultrasonography revealed findings suspicious for malignancy. Answer C: Free T4 is not required if the TSH is normal, as it is in this patient. If the serum TSH is normal or elevated then FNA biopsy is indicated. Answer D: Radionuclide scanning can be used to determine if a nodule is "hot," "warm," or "cold" based on the amount of radioactive isotope uptake. However, thyroid scintigraphy is no longer part of the initial diagnostic management of thyroid nodules in patients with a normal TSH but should be used as the next step in patients in patients with decreased TSH.

A 29-year-old female from Ecuador presents to the emergency room complaining of abdominal pain. The pain is described as diffuse, non-radiating, intermittent and associated with nausea. A chart review shows several emergency visits, most recently last month for a headache associated with numbness and weakness in the right arm. Other visits were significant for an episode of chest pain and recurrent knee pain. She used to be sexually active but states that she has no desire now as she is very worried about her health. In the past she has seen a primary care physician, a gastroenterologist, and a neurologist for her symptoms without relief. She has had multiple prior ultrasounds as well as CT and MRI scans of the abdomen that were unremarkable. What is the most appropriate initial management for this patient? A. acetaminophen B. CT abdomen and pelvis C. fluoxetine D. frequent office visits E. psychiatry consultation

The correct answer is: D Those with somatic symptom disorder must present with multiple physical complains beginning before age of 30 years old. The physical complaints must consist of at least four different sites of pain as present in this patient with abdominal pain, headache, chest pain and knee pain. Review of systems must be positive for at least 2 gastrointestinal symptoms like abdominal pain and nausea; 1 sexual symptom with decreased sexual drive and 1 neurological symptom with numbness/weakness in arm. Also, this patient has gone to multiple specialists with advanced imaging and no clear understanding of the pathophysiology of her symptoms. Therefore, this patient meets all the criteria for a somatization disorder. Although it is important to rule out any significant organic diseases while avoiding elaborate workups, this patient's current non-specific symptoms of abdominal pain and nausea do not require any further imaging at this point. Treatment of somatoform disorders is focused on frequent office visits with the same physician to acknowledge the patients symptoms along with conservative management of any psychiatric complaints like depression or anxiety. Pain can be controlled with over the counter analgesics like acetaminophen or ibuprofen as needed. Pharmacologic therapy is usually started with low dose fluoxetine or other SSRI's and titrated up as needed for management of depression and anxiety. Psychotherapy is also an integral part of management with cognitive behavioral therapy and psychiatrist referral. Although all the answer choices above except imaging are integral part for treatment of somatization disorder, regular scheduled office visits is the most appropriate management for this patient.

A 54-year-old female presents to the clinic for a routine wellness examination. Cardiovascular examination is consistent with normal S1, normal S2 with no murmurs, rubs or gallops. Vitals include a blood pressure of 150/80 mmHg, respiratory rate of 16/min, heart rate of 72/min, and a body mass index of 23 kg/m2. She states she lives an active lifestyle and walks about 2 miles daily. She denies any history of diabetes or high cholesterol. She denies any prior history of smoking or alcohol abuse. This patient would be placed at significant risk for cardiovascular disease if she has a family history of a myocardial infarction in her A. aunt at age 45 B. brother at age 60 C. father at age 55 D. mother at age 65 E. sister at age 60

The correct answer is: E Coronary artery disease (CAD) is the number one cause of morbidity and mortality in the United States of America which makes it a high-yield topic for boards. There are several modifiable and non-modifiable risk factors that place patients at an increase risk for CAD. Modifiable risk factors include hypertension, diabetes, smoking, obesity, hyperlipidemia and inactivity. Since the patient's history is negative for any diabetes, smoking, obesity and the patient states she walks 2 miles daily, she qualifies for one risk factor of hypertension with a SBP of 150 mmHg. Non-modifiable risk factors include age and family history. For women, age greater than 55 years old is a risk factor however this patient is 54 years-old which is not a risk factor. A family history in father or brother with a heart attack before age 55, or in mother or sister before age 65, would make this patient more likely to get heart disease. Therefore, family history of a heart attack in patient's sister at age 60 years old is the correct answer choices as it significantly increases her risk for CAD.

A 40-year-old female is evaluated for bilateral nipple discharge. History is limited due to patient's baseline mental status as she resides in a nursing home due to mental retardation. According to her nursing home records, the patient has no family history of breast cancer and has never been pregnant. Vital signs are all within normal limits. A clinical breast examination confirms the presence of non-bloody, milky bilateral nipple discharge. No palpable mass or discoloration is appreciated. Question 1 of 2 in this set Which of the following diagnostic studies will confirm the diagnosis? A. β-hCG B. contrast-enhanced CT of the head C. mammography D.TSH and T4 E. prolactin

The correct answer is: E Galactorrhea, abnormal lactation, occurs in about 20-25% percent of the women. Estrogen, progesterone, and prolactin are required for lactation to occur whereas dopamine inhibit its release. Differential diagnosis consists of pregnancy, prolactinomas (pituitary adenoma), hypothyroidism, medications and breast stimulation. A step-wise approach to diagnosis of galactorrhea is important with the first step being a β-hCG to rule out pregnancy. The second step is to check for prolactin level, for it is one of the most common causes of galactorrhea. If prolactin levels are normal, the next step is to check for thyroid function tests to rule out hypothyroidism followed by a detailed review of patient's medications. If prolactin level is elevated to greater than 20 ng/mL, the next step is to do an MRI of the brain to evaluate for the size of the prolactinoma. Since the question is asking for the confirmatory test, a prolactin level is most appropriate. Answer A: A β-hCG level should be checked in all female patients who present with galactorrhea. However, since this question asked for the confirmatory test for the most likely diagnosis (prolactinoma), this is not the best answer choice. Answer B: An MRI is the most appropriate study. If the prolactin levels are found to be elevated, the next best step is to check MRI of the brain to assess the size of the prolactinoma. However, a prolactin level must be checked prior to any imaging. Answer C: A mammogram or breast ultrasound is also important part of the diagnostic work up for galactorrhea. Patients with either unilateral discharge or colored discharge like bloody or green discharge must have breast imaging to rule out underlying cancer pathology. However, there is no role for breast imaging yet in a patient with galactorrhea with likely diagnosis of prolactinoma. Answer D: Hypothyroidism is present in less than 10% of the cases of galactorrhea. After ruling out pregnancy and prolactinoma, thyroid function tests must be checked. Common findings include a high TSH is high and low thyroxine. Treatment includes initiating thyroid hormone replacement therapy.

35-year-old teacher at a day care center starts having nausea, vomiting and abdominal pain while at work. Later that week she is diagnosed with hepatitis A. History reveals the day care center takes care of children ages 1 year to 10 years. The most appropriate means of prophylaxis includes A. hand hygiene B. reassurance C. vaccination of children only D. vaccination of all children and staff E. vaccination of all unvaccinated children and staff

The correct answer is: E Hepatitis A is one of the most common preventable infections transmitted via fecal-oral route. Incubation period is about 2 to 6 weeks and people can remain infected about 1 week after being symptomatic as well. Handwashing and chlorination are very important for decreasing the spread of infection; however, post-exposure prophylaxis should be offered to all close contacts as well. Hepatitis A vaccine or Immunoglobulin should be administered to all unvaccinated children and staff of the day care center. Vaccination is recommended for age 12 months to 40 years of age whereas immunoglobulin is recommended in children of age less than 12 months or patients with immunocompromised status or chronic liver disease. Persons who have been vaccinated in the past do not need to receive another dose; therefore, making vaccination of all unvaccinated children and staff the correct answer choice. Answer A: Although hand hygiene is very important in preventing further spread of Hepatitis A virus since the spread is mainly via fecal-oral route. However, it is not as effective as vaccination in preventing Hepatitis A infection post-exposure. Answer B: Reassurance would be inappropriate management since hepatitis A viral infection can spread rapidly and can be a serious infection in toddlers and patients with other co-morbidities. Answer C: Since the staff members are taking care of children at the day care center and changing diapers etc, it is important to vaccinate them as well for infection control. Therefore, in addition to the vaccination of children, any unvaccinated staff members and especially any staff members taking care of toddlers wearing diapers must be vaccinated as well. Answer D: Any children or staff who have already been vaccinated in the past do not need to be re-vaccinated as per CDC guidelines.

A 70-year-old male presents for a routine wellness examination. Social history reveals a 40-pack-year history of cigarettes. Vitals reveal a blood pressure of 154/92 mmHg. Physical examination reveals a pulsatile abdominal mass, and an abdominal ultrasound reveals a 5.0 cm infra-renal abdominal aortic aneurysm. Which of the following is the next appropriate step in the management of this patient's condition? A. control of hypertension B. control of LDL cholesterol C. endovascular aneurysm repair D. open aneurysm repair E. smoking cessation

The correct answer is: E In asymptomatic patients, monitoring is appropriate for lesions < 5.5 cm in size. Small aneurysms (3.0 - 4.0 cm) should be monitored by ultrasound every two years. Aneurysms 4.0 - 5.4 cm should be monitored by ultrasound or CT every six to twelve months. Smoking is a major risk factor for aneurysm formation, growth, and rupture, and smoking cessation is the single most important intervention in patients with non-surgical aortic aneurysms. Answers A & B: Medical management of hypertension and aggressive control of cholesterol have not been shown to impact the size and enlargement of aortic aneurysms to the degree that of smoking cessation. They do however significantly reduce cardiovascular morbidity and mortality and are important interventions in patients at risk for cardiovascular disease. Answers C & D: Indications for surgical repair include (1) lesion size >5.5 cm (2) rapid enlargement of smaller lesions and (3) symptomatic or ruptured aneurysms. Because this patient's lesion is small and asymptomatic, surgical intervention is not indicated at this time.

A 46-year-old male presents for a routine wellness examination. Past medical history is pertinent for liver disease and hypertension treated with furosemide, aldactone and propranolol. The patient reports that he stopped drinking alcohol about 6 months ago after being diagnosed with alcoholic cirrhosis. Review of systems is positive for early satiety and decreased oral intake, but the patient denies any abdominal pain, nausea or vomiting. Physical examination reveals non-tender, moderately distended abdomen with a positive fluid wave. You note paravertebral fullness from T6-9 and a tender nodule in the 6th intercostal space on the right at the midclavicular line. Bilateral lower extremity edema is also noted. Which of the following is the most appropriate recommendation regarding his diet? A. fluid restriction B. high fiber C. low fat D.low protein E. low sodium

The correct answer is: E In patients with cirrhosis with portal hypertension, sodium retention plays a key role in management of ascites. Higher levels of sodium result in retention of water which increases total plasma volume. Normally, vasoconstrictor system responds to the increase in sodium and water retention by excreting excess sodium however this mechanism is impaired in patients with cirrhosis due to liver dysfunction. Decreased excretion of sodium in cirrhosis results in worsening of portal hypertension which results in developments of ascites. Therefore, a negative sodium balance with low sodium intake (< 2 g/day) and high sodium excretion by kidneys via diuretics is key in conservative management of ascites in portal hypertension. In patients who have never been on diuretics before and remain fluid overloaded despite diuretics, dietary non-compliance is the usual etiology of refractory ascites. Therefore, low sodium diet is the most appropriate dietary modification for prevention and management of abdominal ascites and peripheral edema in a patient with cirrhosis. In patient's with liver disease you may see several signs on your structural exam. You may note TART changes from T6-9 as a result of viscerosomatic reflexes from the liver. Additionally you may note an anterior chapman point in the 5th or 6th intercostal space on the right, typically near the midclavicular or midaxillary line. Answer A: Although in practice most patients with cirrhosis and ascites are instructed to limit fluid intake to less than 1000 cc per day, there are no studies showing a significant difference with fluid restriction in these patients. Also, water excretion is usually intact in patients with cirrhosis, as compared to the impaired sodium excretion. Therefore, sodium restriction is more important than fluid restriction in patients with chronic liver cirrhosis. Answer B: High fiber diet plays an important role in management of chronic constipation and diverticulosis. However, there is no importance of management of dietary fiber in patient's with cirrhosis. Answer C: Low fat diets are typically recommended for patients with history of coronary artery disease, dyslipidemia and obesity. There are no specific indications for a low fat diet in patients with cirrhosis. Answer D: A low protein diet (1.2-1.5 g/kg/day) is not typically indicated for cirrhotic patients unless the patient develops hepatic encephalopathy secondary to high level of ammonia or is refractory to management with lactulose. Recommendations include protein restriction of 1.2-1.5 g/kg/day while increasing vegetable protein, dairy protein, and branched chain amino acids. This patient's mental status is intact with no signs of encephalopathy; therefore, a low protein diet is not appropriate at this time.

A 78-year-old male presents for a routine wellness examination with complaints of a chronic productive cough. History reveals a 50 pack-year smoking history, although the patient states he has abstained from smoking ten years ago. Physical examination reveals distant breath sounds without crackles or wheezing. The extremities are non-edematous without clubbing, the oropharynx is pink and moist without lesions, and the abdomen is soft and non-tender without masses. A plain film radiograph of the chest is obtained revealing a depressed diaphragm, increased retrosternal space, and hypovascularity of the lung parenchyma. Which of the following therapies will best prevent mortality and improve the quality of life in this patient? A. influenza and pneumococcal vaccines B. inhaled anticholinergics C. inhaled corticosteroids D. prophylactic antibiotics E. supplemental oxygen

The correct answer is: E Long-term oxygen supplementation is the only treatment for severe COPD that has been shown to improve survival and quality of life, decrease dyspnea scores, and decrease pulmonary artery pressure. Many patients with stable severe COPD have chronic hypoxemia. Improved survival is likely due to improved pulmonary hemodynamics. Improved quality of life is likely secondary to reduced dyspnea during exercise, which allows for better performance of activities of daily living. Those patients who have stable COPD with chronic hypoxemia at rest should be prescribed long-term oxygen therapy. Chronic hypoxemia is defined as PaO2 < 55 mmHg or SpO2 < 80 %. Other important non-pharmacologic treatment includes smoking cessation, as it improves lung function initially and eventually slows annual loss of FEV1. Exercise and pulmonary rehabilitation can decrease pulmonary symptoms and improve exercise tolerance. Pharmacologic management of stable symptomatic COPD includes inhaled bronchodilators (beta agonists and anticholinergics) given alone or in combination with inhaled glucocorticoids depending upon the severity and impact of disease, risk of exacerbations, and response to therapy. Answer A: Indeed, all COPD patients should receive influenza and pneumococcal vaccines according to their respective vaccine schedules. These are both quite important in improving morbidity and mortality. However, neither vaccine can improve pulmonary artery pressure, dyspnea, survival, or quality of life. Answer B: Bronchodilators are the mainstay of pharmacologic treatment for patients with COPD. They can actually induce long-term improvements in symptoms, exercise capacity, and airflow limitation. Inhaled anticholinergics such as ipratropium have fewer side effects and better response than inhaled beta agonists, but intermittent use of a beta agonist can be beneficial as an additional agent. Long-acting beta agonists may be useful, since they last through the night. Tiotropium is a long-acting anticholinergic that should be the bronchodilator of choice. However, these medications cannot improve the measures mentioned in the question stem. Answer C: Inhaled corticosteroids can provide symptomatic relief as symptoms worsen, and oral steroid bursts during exacerbations can also decrease symptoms. Ultimately, they may reduce the airway and systemic inflammation seen in COPD. Inhaled corticosteroids can decrease exacerbations and modestly slow down the progression of respiratory symptoms, but have little impact on lung function and mortality. Ultimately, these medications cannot improve the measures mentioned in the question stem. Answer D: Antibiotic therapy may play a role in patients with chronic bronchitis who have increased volume or a purulent appearance to their sputum. However, these are not generally given in prophylaxis. Chronic antibiotic therapy is generally not indicated for the majority of patients with stable COPD. However, it is important to note that macrolides (azithromycin) can have anti-inflammatory effects beyond their antibiotic effect. Patients who have frequent exacerbations despite maximal pharmacological therapy may benefit from antibiotic prophylaxis with azithromycin. If COPD is associated with bronchiectasis, there may be some benefit to chronic antibiotic therapy. Despite all this though, antibiotics cannot improve the measures asked about in the stem.

A 32-year-old G1P0 female at 16 weeks' gestation presents with vaginal bleeding. History reveals she had pelvic inflammatory disease of unknown etiology prior to conception. Pelvic examination reveals the cervical os to be closed. The most likely diagnosis is A. complete abortion B. incomplete abortion C. inevitable abortion D. partial abortion E. threatened abortion

The correct answer is: E Spontaneous abortions are classified based on where the products of conception are located and how dilated the cervical os is on physical exam. This patient is experiencing a threatened abortion because she has vaginal bleeding, but no products of conception in the vagina and the cervical os is still closed. It is uncertain whether this pregnancy will be viable or whether it will go on to be a complete abortion. Follow-up examination will be required. If on follow-up examination, a spontaneous abortion - incomplete or complete has been diagnosed then the patient can be counseled on her options for management which include: expectant management and observation while awaiting for the products to pass or be reabsorbed without medical intervention, medical management with misoprostol, or a surgical option with dilation and curettage. The risks and benefits of each option should be explained to the patient for her to chose the best management in her situation. The patient should also be given appropriate humanistic care and counseling services for grief management as warranted. Answer A: A complete abortion has occurred when the products of conception have completely evacuated the uterus and cervical canal and the cervical os is now closed. The uterus will be small and contracted. The patient may continue to experience a small amount of vaginal bleeding and/or crampy pelvic pain. Answer B: An incomplete abortion has occurred when the patient is experiencing vaginal bleeding and cramping pelvic pain, the cervical os is open and the products of conception can be seen or palpated within the cervical canal. Answer C: An inevitable abortion is occurring when a patient has vaginal bleeding and often cramping pelvic pain accompanied by a dilated cervical os where the products of conception can be seen or palpated through the internal cervical os. Answer D: Partial abortion is not an appropriate or defined medical term.

A 89-year-old female nursing home resident presents for follow-up of her recurrent nausea, vomiting, and loose stools. She has undergone extensive testing but her providers have been unable to determine an etiology. She has been managing her symptoms with metoclopramide for over one year. Her past medical history is pertinent for hypertension and hyperlipidemia. Today, she remarks that her nausea and diarrhea are improved, but on physical examination she is observed to be constantly smacking her lips and jerking her head. This finding is called A. akathisia B. chorea C. dystonia D.secondary parkinsonism E. tardive dyskinesia

The correct answer is: E Tardive dyskinesia (TD) is a hyperkinetic movement disorder that appears with a delayed onset after prolonged use of dopamine receptor blocking agents. TD can present with chorea, athetosis, dystonia, akathisia, stereotyped behaviors and rarely, tremor. The term "tardive" differentiates these dyskinesias from acute dyskinesia, parkinsonism, and akathisia, which appear very soon after exposure to antipsychotic drugs. TD can include a variable mixture of orofacial dyskinesia, athetosis, dystonia, chorea, tics, and facial grimacing. The symptoms involve the mouth, tongue, face, trunk, or extremities. Oral, facial, and lingual dyskinesia are especially conspicuous in elderly patients. They may include protruding and twisting movements of the tongue, smacking movements of the lips, retraction of the corners of the mouth, bulging of the cheeks, or chewing movements. The onset of TD is insidious and typically occurs while the patient is receiving an antipsychotic drug. It may appear as early as one to six months following drug exposure, though it was recently thought to occur only after two or more years of treatment. Though the diagnosis is relatively straightforward, it must be differentiated from schizophrenia and Wilson's disease, as well as other antipsychotic drug induced extrapyramidal syndromes that often coexist. In this patient, we observe lip smacking and head jerking, which are classic for tardive dyskinesia. Other agents that commonly cause TD include neuroleptics, older atypical antipsychotics, metoclopramide (as in this patient), antihistamines, and fluoxetine.

A foreign medical graduate from India presents with a cough. History reveals that he previously received the BCG vaccine as a child. Which of the following would indicate an infection with tuberculosis in this patient? A. 5 mm PPD B. 10 mm PPD C. 15 mm PPD D. negative QuantiFERON-TB Gold E. positive QuantiFERON-TB Gold

The correct answer is: E The immigrant has two risk factors for possible TB infection: he is from India which has a high prevalence of TB and works in a high-risk congregate setting. Therefore, when he presents with a cough TB must remain on the differential. He received the BCG vaccine as a child which can cause a false positive screen on the PPD. Therefore the preferred test is the QuantiFERON-TB Gold test. A positive QuantiFERON-TB Gold will indicate a TB infection in this patient. The QuantiFERON-TB Gold is a blood test know as an interferon gamma release assay. It tests the patient's immune system for its response to Mycobacterium tuberculosis. It is the preferred test in patients who have been previously vaccinated with BCG or who have difficulty returning to have the PPD test read. Answer A: 5 mm of induration on a PPD test indicates a positive screen in persons who: are infected with HIV; recently had contact with a person infected with TB; fibrotic changes on chest radiograph consistent with TB; have an organ transplant; are immunosuppressed. The foreign medical graduate from India with a cough does not fit any of these categories. Thus a 5 mm induration would not indicate a positive screen. Without a positive screen no further testing is recommended. Answer B: 10 mm of induration is a positive screen in persons who: are recent (<5 years) immigrants from high prevalence countries; are injection drug users; are residents or employees of high-risk congregate settings such as prisons, military barracks, nursing homes, or hospitals; are mycobacteriology laboratory personnel; have a clinical condition that places them at high risk; are under the age of 4; are an infant, child or adolescent exposed to an adult in a high-risk category. The foreign medical graduate from India falls into this category for two reasons- he's a recent immigrant and he works in a high-risk congregate setting. This would indicate a positive screen with 10mm of induration on the PPD test. However, he has a history of BCG vaccination which can cause there to be a false positive screen. Therefore it is more appropriate to test with QuantiFERON-TB Gold. Answer C: 15 mm of induration on a PPD test is a positive screening test on all persons, even those without risk factors for TB infection. Therefore 15 mm of induration would also be a positive test in this patient; however, since he's received the BCG vaccination which can cause a false positive it is more appropriate to test with the QuantiFERON-TB Gold. Answer D: The majority of healthy adults who have negative QFT-G results are unlikely to have M. tuberculosis infection and do not require further evaluation.

A 21-year-old G1P0 woman from Oklahoma presents at 12 weeks' gestation for routine prenatal care. Her past medical history is unremarkable. What is the most appropriate initial prenatal screening test at this time? A. fasting blood glucose B. GBS swab C. HCV antibody D. PPD E. urine culture

The correct answer is: E The most appropriate initial screening test at 11 weeks' gestation is a urine culture to rule out asymptomatic bacteriuria. According to the USPSTF, asymptomatic bacteriuria should be screened for between 12-16 weeks gestation or at the first prenatal visit if it is later than 16 weeks. It is important to screen for asymptomatic bacteriuria and treat it to decrease the risk of early pregnancy loss and pre-term labor. Answer A: Fast blood glucose is an appropriate screening test for gestational diabetes. The United States Preventative Services Task Force (USPSTF) and the American College of Obstetrics and Gynecology recommend it be done between 24 and 28 weeks gestation. Furthermore the USPSTF states that there is insufficient evidence to recommend screening prior to 24 weeks gestation. Answer B: A GBS swab screens for Group B streptococcal colonization in a pregnant female that increases the risk for Group B streptococcal infections in the neonate. It is recommended to screen pregnant women between 35-36 weeks gestation as the screening test is good for 5 weeks. If the GBS swab returns a positive culture it is recommended that the pregnant female receive antibiotic prophylaxis during labor to reduce the risk of transmission of Group B streptococcus to the neonate when passing through the birth canal. Answer C: There is no recommendation for routine Hepatitis C virus antibody screening in pregnant women. It is recommended to screen for hepatitis C in high risk populations such as IV drug users. If a mother is positive she should receive counseling and appropriate treatment. Approximately 6 in 100 infants born to HCV positive mothers become infected with Hepatitis C. No change in delivery methods has been shown to reduce the risk of transmission. Answer D: There is no universal recommendation to screen for tuberculosis in all pregnant females. It is however recommended to screen in high risk populations such as: HIV+, immunocompromised, symptoms of TB (fever, night sweats, cough, and weight loss), those from Latin America, the Caribbean, Africa, Asia, Eastern Europe, and Russia; and those who live or work somewhere in the United States where TB disease is more common (homeless shelters, prison or jails, or some nursing homes). Pregnant women who test positive for TB should be treated for latent TB as soon as diagnosed during pregnancy. Isoniazid is the prefered treatment in pregnant women. Answer E: The most appropriate initial screening test at 11 weeks gestation is a urine culture to rule out asymptomatic bacteruria. According to the USPSTF, asymptomatic bacteruria should be screened for between 12-16 weeks gestation or at the first prenatal visit if it is later than 16 weeks. It is important to screen for asymptomatic bacteruria and treat it to decrease the risk of early pregnancy loss and pre-term labor.

For each numbered item (patient presentation), select the one heading (treatment) most closely associated with it. Each lettered heading may be selected once, more than once, or not at all A. citalopram B. donepezil C. memantine D. omega-3 fatty acids E. selegiline A 74-year-old female presents to her primary care physician with complaints of slowed movements and joint stiffness. She never had tremors before but now has a tremor, which began three months ago, when she is resting which goes away if she engages in a physical task. Her husband notes that she has been having some difficulties with cognitive thinking, has difficulty with balance, and problems with getting up from the chair. On physical examination, it is noted that she is walking with small shuffling steps and has a stooped posture. Her past medical history is significant for COPD and hypertension, for which she takes medications. She has no past psychiatric history.

The correct answer is: E This patient demonstrates classic motor and cognitive symptoms suggestive of Parkinson's disease. Selegiline is a monoamine oxidase inhibitor (MAOI) used to treat Parkinson's disease, inhibiting monoamine oxidase B activity. Monoamine oxidase metabolizes dopamine and phenylethylamine. It is often used as an adjunctive agent and reduces the metabolism of levodopa. Due to its ability to increase the bioavailability of dopamine, this drug has also been shown to be effective in the treatment of depression and dementia. Adverse effects include hypertensive crisis, weight gain, anxiety and insomnia. Patients who take a monoamine oxidase inhibitor should be placed on a low tyramine diet.

A 47-year-old postmenopausal female falls while carrying groceries into her house and sustains a distal radial fracture. A chemistry panel reveals a calcium level of 11.2 mg/dL and her albumin level is within normal limits. Upon further investigation, the patient is found to have a parathyroid hormone level of 200 pg/mL. She is currently not taking any prescribed or over the counter medications. Which of the following is the best course of action for this patient? A. bisphosphonate therapy B.elimination of vitamin D and calcium from diet C. estrogen replacement therapy D. furosemide and intravenous saline E. parathyroidectomy

The correct answer is: E This patient has primary hyperparathyroidism, as determined by the elevated serum calcium level and elevated parathyroid hormone. Parathyroidectomy is a safe and effective treatment for primary parathyroidism, with cure rates of 95-98% and complication rates of 1-3%. Furthermore, because this patient has serum calcium level that is below 12 mg/dL, she can proceed to surgery without further steps to premedicate her (i.e. loop diuretics with IV saline) to reduce the serum calcium level prior to surgery. Newer surgical protocols have attempted parathyroid gland sparing techniques, in which the abnormal gland is detected pre-operatively and is selectively removed, leaving the normal functioning native tissue. Complications include recurrent laryngeal nerve palsy and post-operative hypocalcemia, but these are becoming less frequent with minimally invasive techniques. Thorough history should be gathered, including the use of thiazide diuretics or lithium as they can cause elevations in serum calcium and parathyroid hormone. Answers A & D: In the rare patients with higher calcium levels (>12 mg/dL), preoperative treatment is required with saline infusion, followed by loop diuretics, and intravenous bisphosphonates or calcimimetics (cinacalcet) to reduce the serum calcium level and minimize the risk of complications associated with more severe hypercalcemia. These complications include hypovolemia, acid−base abnormalities, and cardiac-rhythm disturbances. Answer B: If a patient were to decline surgical removal of the parathyroid glands for the treatment of primary hyperparathyroidism, then they should be followed clinically and monitored for osteopenia and osteoporosis. Adequate dietary intake of calcium and vitamin D (if they are deficient) should be provided to decrease the resorption of bone as much as possible. It would be detrimental to the patient to withhold dietary calcium and vitamin D in these patients. Answer C: The patient's fracture from minor trauma is most likely related to her primary hyperparathyroidism, not her postmenopausal state. It is true that postmenopausal physiology can lead to increased susceptibility of fractures and bone demineralization; however, estrogen replacement therapy will not address the abnormality of the patient's parathyroid glands.

A 55-year-old man presents for follow-up regarding his hypertension. He reports intermittent headaches that come and go without treatment. His other medical problems include hypercholesterolemia, diabetes, and intermittent claudication. He has smoked one pack of cigarettes per day for the last 30 years. His medications include aspirin, metoprolol, hydrochlorothiazide, enalapril, amlodipine, simvastatin, metformin, and glyburide. His temperature is 37°C (99.5°F), pulse is 85/min, respirations are 16/min, and blood pressure is 170/100 mmHg. Which of the following findings would be most likely? A. elevated aldosterone:renin ratio B. increased urinary vanillylmadelic acid C.intercostal artery pulsation D. positive urine drug screen E. right sided periumbilical bruit

The correct answer is: E This patient is at high risk for atherosclerosis and is already manifesting with peripheral vascular disease. His hypercholesterolemia, smoking, and diabetes all place him at high risk. Although he is being treated with 4 medications for hypertension: metoprolol, hydrochlorothiazide, amlodipine, and enalapril, he is still hypertensive. Resistant hypertension in a patient at high risk for atherosclerosis makes renal artery stenosis a very likely cause. A bruit would be auscultated on the side of the stenosis. Definite diagnosis requires a renal MRI or CT angiography of the renal arteries. Treatment requires control of his atherosclerotic risk factors, lifestyle modification, and renal artery stenting. The ACE-I should be discontinued since it can exacerbate renal impairment in patients with stenosis. Answer A: Patients with high aldosterone: renin ratios have primary hyperaldosteronism (Conn's syndrome) and are at risk for hypernatremia and hypokalemia. Answer B: Pheochromocytoma, a tumor of the adrenal medulla, will produce excessive catecholamines causing hypertension and showing the catecholamine end product vanillylmadelic acid (VMA) in the urine. This patient has a high risk for atherosclerosis. Answer C: Patients with coarctation of the aorta will show intercostal artery pulsations due to shunting of blood around the coarctation through the intercostal arteries. Coarctation is usually diagnosed much earlier than 55. Answer D: A urine drug screen could reveal an illicit substance such as cocaine or methamphetamines. However, this patient is at high risk for artery stenosis, making that diagnosis more likely.

A 45-year-old male presents with knee pain following an injury after jumping out of a burning building from the second floor. History reveals that he heard a loud popping sound after landing, and now his knee is swollen and gives out on him while standing. Physical examination reveals tenderness to palpation above the superior pole of the patella. The patient is unable to perform a straight leg raise. The most likely cause is a/an A. Achilles tendon rupture B. grade II anterior cruciate ligament sprain C. patella dislocation D. popliteus tendinitis E. rectus femoris rupture

The correct answer is: E This patient is presenting with a quadriceps tendon tear. This most commonly occurs in patients older than 40-years of age, and is more common in males. A quadriceps tendon tear results form eccentric loading of the knee extensor mechanism, more often when the foot is planted and the knee is bent. The rectus femoris muscle is part of the quadriceps tendon and originates on the anterior inferior iliac spine and inserts onto the superior pole of the patella to form the central portion of quadriceps tendon. Innervation is from the femoral nerve. The rectus femoris acts to extend the knee. Patients with a ruptured rectus femoris muscle will have anterior knee pain and tenderness to palpation at the defect. Physical examination will reveal a palpable defect above the superior pole of the patella. A quadriceps tendon tear will disrupt the extensor mechanism of the knee. Thus, the patient will be unable to perform a straight leg raise. Answer A: The Achilles tendon is formed by the confluence of the soleus muscle tendon along with the medial and lateral gastrocnemius tendons onto the calcaneus. An Achilles tendon rupture usually occurs in younger athletes, usually around the age of 30 to 40-years-old. Most ruptures occur after a traumatic injury during a sporting event. Rupture can occur with sudden forced plantar flexion or with violent dorsiflexion in a plantar flexed foot. Patients typically report a "pop" and have difficulty with walking and pain in the heel. Patients may have a palpable gap on physical examination, along with a positive Thompson test. The Thompson test is lack of plantar flexion when the calf is squeezed. Treatment is either non-operative or surgical, depending on age and activity level of the patient. Answer B: The anterior cruciate ligament (ACL) prevents excess anterior tibial translation of the tibia on the femur. An ACL tear commonly occurs with a twisting mechanism when the knee is hyperextended. Thus, it commonly occurs in sports like football, basketball, and gymnastics. ACL tears are more common in women. Female athletes have a tendency to increase their quadriceps strength disproportionately to their hamstring strength, thus putting them at increased risk for an ACL injury. Imbalanced or excessive knee extensor strength causes abnormal tension on the ACL. Strengthening exercises of the hamstrings protects against this phenomenon. An ACL tear will present with swelling and a hemarthrosis within one hour of the event. The patient will also be unable to bear weight. The patient will have a positive anterior drawer test and a positive anterior Lachman's test. Treatment requires an ACL reconstruction surgery. A grade 1 tear is a slightly damaged ligament, a grade 2 tear is a partial tear, and a grade 3 tear is a complete tear. Answer C: An acute patella dislocation most commonly results from a non-contact twisting injury with the knee extended and foot externally rotated. It may also result from a direct blow to the knee. Risk factors include Ehlers-Danlos syndrome, dysplastic vastus medialis oblique muscle, femoral trochlear dysplasia, increased quadriceps angle, femoral anteversion, genu valgum, and external tibial torsion. Patients will present with a large hemarthrosis, medial sided tenderness, increased patellar translation, and patellar apprehension. That is, a passive lateral translation results in guarding and sense of apprehension. First step would be to order knee radiographs. Answer D: Popliteus tendinitis is inflammation of the tendon. The popliteus muscle originates on the lateral surface of the lateral femoral condyle and inserts onto the posterior surface of the proximal tibia. It acts to internally rotate the tibia and unlock the knee during knee flexion. It is innervated by the tibial nerve. Patients will present with posterior and lateral knee pain that is worsened with standing on a slightly flexed knee or with rotation of the knee. It is an overuse syndrome, and treatment is with activity modification, stretching, NSAIDs, and ice.

A 7-year-old male presents with a temperature of 38.4ºC (101.2ºF) and urinary incontinence. History reveals two previous urinary tract infections. He appears ill, has a positive Lloyd's sign on the right and suprapubic tenderness on palpation. A urinalysis is obtained and reveals 1-2+ RBCs and is leukocyte esterase positive. The most appropriate modality to evaluate the underlying cause is A. BUN and creatinine B. CT of the abdomen and pelvis C. cystoscopy D. renal ultrasound E. voiding urethrogram

The correct answer is: E This patient is presenting with febrile urinary tract infection based on his temperature of 38.4ºC (101.2ºF), physical exam findings and suggestive urinalysis. Lloyd's sign is positive costovertebral tenderness significant for pyelonephritis. New onset urinary incontinence, back pain and abdominal pain are symptoms suggestive of urinary tract infection in older children. Leukocyte esterase is 94% sensitive and 72% specific for diagnosis of urinary tract infection in a patient with high suspicion for urinary tract infection. The American Academy of Pediatrics recommends a voiding cystourethrogram after second febrile urinary tract infection to evaluate for possible vesicoureteral reflux. Vesicoureteral reflux is a congenital defect in which urine flows backwards from the bladder into the kidneys. The reflux of urine back into the kidneys transfers bacteria from the bladder into the upper urinary tract causing infection. If not diagnosed and corrected vesicoureteral reflux may lead to irreversible damage to the kidneys. A voiding cystourethrogram is radiologic procedure done by injecting contrast into the bladder and then taking x-ray images of the bladder during voiding to determine if there is reflux from the bladder into the ureters. Answer A: BUN and creatinine are elevated in a patient with acute kidney injury which may be due to febrile urinary tract infection such as pyelonephritis, but its elevation is nonspecific and not indicative of the cause. A voiding urethrogram is a better study in this case because it can identify a cause such as vesicoureteral reflux. Answer B: CT abdomen and pelvis is not the imaging modality of choice to diagnose vesicoureteral reflux which may be the underlying cause of the febrile UTI. A CT abdomen and pelvis may be helpful in determining if the patient had a pyelonephritis, but the CT would not indicate an underlying cause for the UTI. Answer C: Cystoscopy is endoscopy of the urinary bladder via the urethra, usually done under general anesthesia when performed in children. This procedure is not first line because it is invasive with higher risk of adverse events in children. It is generally reserved (in children) to better evaluate structural defects with the expectation of surgical correction such as stent insertion, ureteral dilatation, etc. Answer D: Infants and young children 2-24 months of age with initial febrile urinary tract infection should undergo a renal and bladder ultrasound to defect underlying anatomical abnormalities that may be contributing to development of urinary tract infection. If abnormality such as hydronephrosis is noted on ultrasound, a voiding urethrogram should then be obtained. However, recent evidence indicates that renal ultrasound may miss some vesicoureteral reflux and therefore a voiding urethrogram is indicated when a second febrile urinary tract infection occurs.

A 52-year-old male presents with a bitter taste in his mouth and burning in his chest that occurs after heavy meals. The symptoms began approximately 1 year ago and have been worsening over the past several months with new-onset dysphagia over the past two months. Past medical history includes hypertension treated with hydrochlorothiazide and a 25-pack-year smoking history. Physical examination reveals no chest wall tenderness with clear lungs and a heart that has a regular rate and rhythm without murmurs. The abdomen is soft without tenderness or distention. Which of the following is the most appropriate management? A. barium swallow test B. esophageal manometry C.lifestyle modifications D. proton pump inhibitor therapy E. upper endoscopy

The correct answer is: E This patient is presenting with symptoms of gastroesophageal reflux disease (GERD) as well as new onset dysphagia which is considered an ALARM SYMPTOM making this a case of complicated GERD. Other alarm symptoms include weight loss, bleeding, and iron deficiency anemia. Upper endoscopy is indicated in all patients with alarm symptoms, or complicated GERD. Endoscopy is necessary in order to visualize the underlying pathology presumably causing the alarm symptoms, such as peptic stricture, gastritis, esophageal adenocarcinoma, etc. In this patient, dysphagia from GERD is often associated with esophagitis, which can lead to peptic stricture or Barrett's metaplasia. Upper endoscopy is also indicated if patient who fail to have controlled symptoms following an empiric trail with acid suppressing therapy. Answer A: Performing a barium swallow study prior to an EGD is controversial. A barium swallow test should immediately be performed if there is a history of prior radiation, caustic injury, surgery for laryngeal/esophageal cancer, or signs of a complex stricture. If an EGD is performed and is normal and the patient still has dysphagia to solids a barium swallow should then be performed. If an EGD is normal and the patient has dysphagia to liquids AND solids then esophageal manometry should be performed. Answer B: Esophageal manometry is a second line test that should be performed after an EGD is determined as normal and if the patient also has dysphagia to liquids. It would be beneficial if looking for a specific diagnosis such as achalasia, which this patient is not exhibiting. Answer C: Lifestyle modifications are an important part of symptom management in GERD and all patients should be counseled on them. Lifestyle modifications include elevation of the head of the bed, avoidance of reflux inducing foods (chocolate, peppermint, fatty foods, etc), avoidance of tight-fitting clothes, avoidance of meals before bedtime, decrease in alcohol use, smoking cessation, etc. While this patient should be educated on the appropriate modifications, it is first more essential to perform and EGD given his dysphagia. Answer D: The current recommended therapy for GERD includes a step-up approach to treatment depending upon the severity of symptoms. H2 blockers as needed, along with lifestyle and dietary modifications, are indicated for those with mild symptoms. From there, one can build up to scheduled dosing of prescription H2 blockers, intermittent use of PPI, and eventually, scheduled PPI dosing. This patient will need pharmacologic therapy, but an upper endoscopy needs to be completed first in order to cater therapy.


Kaugnay na mga set ng pag-aaral

Chapter 14: Global Financial Management

View Set

2B - Market Influences on Business

View Set

Cellular Basis of Animal Behavior - Exam 2

View Set

Chapter 9: Constructing Gender and Sexuality

View Set

The Subjunctive in Adverbial Clauses

View Set

4.9 Use remote access technologies

View Set